LSAT 2016

PART II LSAT Practice Tests

LSAT Practice Test 2

You can also take this practice test on your tablet or smartphone as well as your laptop or home computer. See page 7A of the Welcome insert for more information.

Answer Sheet

Directions: Before beginning the test, photocopy this answer sheet or remove it from the book. Mark your answer to each question by filling in the corresponding answer oval in the columns below. If a section has fewer questions than answer spaces, leave the extra spaces blank.

Section I

1. Images

2. Images

3. Images

4. Images

5. Images

6. Images

7. Images

8. Images

9. Images

10. Images

11. Images

12. Images

13. Images

14. Images

15. Images

16. Images

17. Images

18. Images

19. Images

20. Images

21. Images

22. Images

23. Images

24. Images

25. Images

26. Images

27. Images

28. Images

29. Images

30. Images

Section II

1. Images

2. Images

3. Images

4. Images

5. Images

6. Images

7. Images

8. Images

9. Images

10. Images

11. Images

12. Images

13. Images

14. Images

15. Images

16. Images

17. Images

18. Images

19. Images

20. Images

21. Images

22. Images

23. Images

24. Images

25. Images

26. Images

27. Images

28. Images

29. Images

30. Images

Section III

1. Images

2. Images

3. Images

4. Images

5. Images

6. Images

7. Images

8. Images

9. Images

10. Images

11. Images

12. Images

13. Images

14. Images

15. Images

16. Images

17. Images

18. Images

19. Images

20. Images

21. Images

22. Images

23. Images

24. Images

25. Images

26. Images

27. Images

28. Images

29. Images

30. Images

Section IV

1. Images

2. Images

3. Images

4. Images

5. Images

6. Images

7. Images

8. Images

9. Images

10. Images

11. Images

12. Images

13. Images

14. Images

15. Images

16. Images

17. Images

18. Images

19. Images

20. Images

21. Images

22. Images

23. Images

24. Images

25. Images

26. Images

27. Images

28. Images

29. Images

30. Images

SECTION I

Time—35 minutes

25 Questions

Directions: Each group of questions in this section is based on a set of conditions. In answering some of the questions, it may be useful to draw a rough diagram. Choose the response that most accurately and completely answers each question and blacken the corresponding space on your answer sheet.

Questions 1–5

From among eight candidates, an epidemiological think tank must choose four medical scientists for a special research project. Four of the candidates—Bosch, Cristof, Eber, and Gross—are published scientists, and four—Hoff, Jackson, Klipp, and Mann—are unpublished scientists. Bosch, Hoff, Klipp, and Mann are microbiologists whereas Cristof, Eber, Gross, and Jackson are immunologists. The scientists must be selected according to the following conditions:

Exactly two published scientists and two unpublished scientists are selected.

Exactly two microbiologists and two immunologists are selected.

Either Klipp or Gross or both are selected.

1. Which one of the following is an acceptable selection of the scientists for the research project?

(A) Cristof, Gross, Hoff, and Mann

(B) Eber, Gross, Jackson, and Mann

(C) Bosch, Cristof, Jackson, and Mann

(D) Bosch, Gross, Hoff, and Klipp

(E) Bosch, Hoff, Jackson, and Klipp

2. If Bosch and Klipp are selected for the research project, the other two scientists selected must be

(A) two immunologists, both of whom are published scientists

(B) two immunologists, both of whom are unpublished scientists

(C) an immunologist who is a published scientist and an immunologist who is an unpublished scientist

(D) an immunologist who is a published scientist and a microbiologist who is an unpublished scientist

(E) an immunologist and a microbiologist, both of whom are published scientists

3. If Bosch and Cristof are selected for the research project, which one of the following must also be selected?

(A) Hoff

(B) Eber

(C) Gross

(D) Mann

(E) Jackson

4. If Hoff and Mann are selected for the research project, which one of the following could be, but need not be, selected for the research project?

(A) Klipp

(B) Bosch

(C) Jackson

(D) Cristof

(E) Gross

5. If Jackson is selected for the research project, which one of the following must also be selected?

(A) Gross

(B) Bosch

(C) Mann

(D) Cristof

(E) Hoff

Questions 6–12

A tour operator has chosen six stops along the route of the tour—J, K, L, M, N, and P—each with at least one of the following features: scenic views, a historic landmark, and restroom facilities. No stop has any other features. The following conditions must apply:

K has scenic views and restroom facilities.

L has scenic views and a historic landmark.

L and N have no features in common.

M has more features than L.

K and P have exactly one feature in common.

J has fewer features than P.

6. For exactly how many of the stops is it possible to determine exactly which features each one has?

(A) six

(B) five

(C) four

(D) three

(E) two

7. Which one of the following must be false?

(A) Exactly four of the six stops have restroom facilities.

(B) Exactly four of the six stops have scenic views.

(C) Exactly four of the six stops have historic landmarks.

(D) Exactly five of the six stops have restroom facilities.

(E) Exactly five of the six stops have historic landmarks.

8. If all the stops that have historic landmarks also have scenic views, which one of the following must be false?

(A) P has scenic views.

(B) P has restroom facilities.

(C) K has scenic views.

(D) J has scenic views.

(E) J has restroom facilities.

9. If P has no features in common with J but has at least one feature in common with every other stop, then which of the following must be false?

(A) Exactly four of six stops have scenic views.

(B) Exactly four of six stops have restroom facilities.

(C) Exactly four of six stops have historic landmarks.

(D) J has scenic views.

(E) P has restroom facilities.

10. Suppose that no two stops have exactly the same features as one another. In that case, each of the following could be true EXCEPT:

(A) Exactly four of the six stops have restroom facilities.

(B) Exactly three of the six stops have restroom facilities.

(C) Exactly four of the six stops have historic landmarks.

(D) Exactly three of the six stops have scenic views.

(E) Exactly four of the six stops have scenic views.

11. If exactly four of the six stops have historic landmarks, and exactly four of the six locations have scenic views, then each of the following must be true EXCEPT:

(A) N and P have no features in common.

(B) J and K have no features in common.

(C) J and N have no features in common.

(D) J and P have exactly one feature in common.

(E) L and P have exactly one feature in common.

12. Suppose that the condition requiring that M has more features than L is replaced by a new condition requiring that M and L have exactly two features in common. If all of the other original conditions remain in effect, which one of the following must be false?

(A) J and M have no features in common.

(B) M and P have exactly two features in common.

(C) M and P have no features in common.

(D) K and M have exactly one feature in common.

(E) K and M have exactly two features in common.

Questions 13–18

Exactly seven different buses are arriving at Bellevert Station from Delbert, Evans, Fortman, Hampton, Janistown, Koenig, and Lynnville. The following conditions govern their arrivals:

The buses arrive one at a time.

Either the Lynnville or the Koenig arrives fourth.

The Fortman arrives at some time after the Koenig but at some time before the Lynnville.

Both the Hampton and the Janistown arrive at some time after the Evans.

The Hampton does not arrive next after the Janistown; nor does the Janistown arrive next after the Hampton.

13. Which one of the following could be the order in which the buses arrive, from first to last?

(A) Delbert, Evans, Koenig, Fortman, Janistown, Lynnville, Hampton

(B) Hampton, Evans, Delbert, Koenig, Fortman, Lynnville, Janistown

(C) Evans, Hampton, Janistown, Koenig, Fortman, Lynnville, Delbert

(D) Evans, Hampton, Delbert, Koenig, Fortman, Lynnville, Janistown

(E) Evans, Koenig, Delbert, Lynnville, Hampton, Fortman, Janistown

14. If the Koenig arrives at some time before the Evans, then exactly how many different orders are there in which the seven buses could arrive?

(A) four

(B) five

(C) six

(D) seven

(E) eight

15. Which one of the following must be true?

(A) The first bus to arrive is from Evans.

(B) The Koenig arrives at some time before the Lynnville.

(C) The Delbert arrives at some time before the Fortman.

(D) The Janistown arrives at some time before the Lynnville.

(E) The Evans arrives at some time before the Koenig.

16. Which one of the following could be true?

(A) The Delbert is the next bus after the Koenig.

(B) The Delbert is the next bus after the Fortman.

(C) The Fortman is the next bus after the Delbert.

(D) The Evans is the next bus after the Hampton.

(E) The Evans is the next bus after the Fortman.

17. If exactly one of the buses arrives after the Koenig but before the Lynnville, then which one of the following could be true?

(A) The first bus to arrive is the Evans.

(B) The second bus to arrive is the Fortman.

(C) The third bus to arrive is the Evans.

(D) The sixth bus to arrive is the Fortman.

(E) The sixth bus to arrive is the Hampton.

18. If the Delbert arrives at some time before the Evans, then the Koenig must arrive

(A) sixth

(B) fifth

(C) fourth

(D) third

(E) second

Questions 19–25

Exactly seven cargo trucks—P, Q, R, T, U, W, and Z—are to be unloaded at seven warehouse docks, exactly one cargo truck to a dock. The seven docks are side by side and numbered consecutively 1 through 7. Assignment of cargo trucks to docks must meet the following conditions:

P is unloaded at a lower numbered dock than T.

W is unloaded at the dock numbered one lower than the dock at which Q is unloaded.

R is unloaded at dock 1 or else dock 7.

Z is unloaded at dock 4.

19. Which one of the following is an acceptable assignment of cargo trucks to warehouse docks, in order from dock 1 through dock 7?

(A) R, W, Q, Z, T, P, U

(B) R, U, Z, W, Q, P, T

(C) P, U, T, Z, W, Q, R

(D) P, W, U, Z, Q, T, R

(E) P, T, U, Z, R, W, Q

20. It must be true that the lowest numbered dock on which

(A) P can be unloaded is dock 2

(B) Q can be unloaded is dock 3

(C) T can be unloaded is dock 2

(D) U can be unloaded is dock 3

(E) W can be unloaded is dock 2

21. If U is unloaded on dock 5, which one of the following is a pair of trucks that could be unloaded, not necessarily in the order given, at docks whose numbers are consecutive to each other?

(A) R, T

(B) T, W

(C) W, Z

(D) Q, R

(E) Q, T

22. There can be at most how many docks between the dock at which P is unloaded and the dock at which T is unloaded?

(A) five

(B) four

(C) three

(D) two

(E) one

23. If U is unloaded at dock 2, which one of the following must be true?

(A) Z is unloaded at a lower numbered dock than T.

(B) W is unloaded at a lower numbered dock than T.

(C) T is unloaded at a lower numbered dock than R.

(D) P is unloaded at a lower numbered dock than Z.

(E) R is unloaded at a lower numbered dock than Q.

24. If Q is unloaded at a dock numbered one less than the dock at which P is unloaded, then which one of the following must be true?

(A) W is unloaded at dock 6.

(B) U is unloaded at dock 5.

(C) R is unloaded at dock 1.

(D) Q is unloaded at dock 5.

(E) P is unloaded at dock 3.

25. If W is unloaded at dock 1, which one of the following could be true?

(A) U is unloaded at a dock numbered one lower than the dock at which Q is unloaded.

(B) Q is unloaded at a dock numbered one lower than the dock at which U is unloaded.

(C) Q is unloaded at a dock numbered one lower than the dock at which T is unloaded.

(D) P is unloaded at a dock numbered one lower than the dock at which U is unloaded.

(E) P is unloaded at a dock numbered one lower than the dock at which R is unloaded.

STOP

IF YOU FINISH BEFORE TIME RUNS OUT, CHECK YOUR WORK ON THIS SECTION ONLY. DO NOT GO ON TO ANY OTHER TEST SECTION.

SECTION II

Time—35 minutes

25 Questions

Directions: The questions in this section are based on the reasoning contained in brief statements or passages. For some questions, more than one of the choices could conceivably answer the question. However, you are to choose the best answer; that is, the response that most accurately and completely answers the question. You should not make assumptions that are by commonsense standards implausible, superfluous, or incompatible with the passage. After you have chosen the best answer, blacken the corresponding space on your answer sheet.

1. David:   The only way for a professional athlete to be successful after having suffered a serious injury is to relearn how he originally played the game before the injury. It is futile for an athlete to learn a whole new way to play the game.

Kathy:   Wrong. Sam Daughton was a professional quarterback who suffered a serious leg injury and had to go through significant physical therapy and training. When he returned to the game he had left his older, more physical style of play behind and embraced a more successful style of finesse.

Kathy uses which one of the following argumentative techniques in countering David’s argument?

(A) She establishes a solution by excluding the only plausible alternative to that solution.

(B) She offers a developed and relevant analogy that supports his claim.

(C) She undermines his claim by showing that it rests on an equivocation.

(D) She presents an example that counters his claim.

(E) She offers a different explanation for a phenomenon.

Questions 2–3

Digital camera manufacturers typically advertise their products by citing the resolutions of their cameras, usually measured in pixels and indicating the degree of detail at which the camera’s sensor is capable of recording the image in memory. Differences between cameras in this respect are irrelevant, however, since all modern sensors record far more detail into memory than can be represented on electronic screens or printed on photographic paper.

2. Which one of the following most accurately states the main point of the argument?

(A) Advertised differences among cameras in the resolution of their sensors have no practical bearing on the cameras’ relative quality as photographic tools.

(B) Digital camera manufacturers should concentrate on incorporating other desirable qualities into their cameras, rather than concentrating only on the sensors’ resolution.

(C) By concentrating their advertising on the issue of resolution, manufacturers are making a mistake about the interests of potential purchasers of cameras.

(D) Differences among electronic screens and photographic paper have a more significant effect on the quality of the image than do differences in the resolution of digital camera sensors.

(E) Apart from differences in resolution, there is no practical difference among modern digital cameras in the quality of images they produce.

3. The argument depends on assuming which one of the following?

(A) In determining the amount of detail reproduced in the photographic image, differences in the resolutions of available cameras neither compound nor decrease the deficiencies of available electronic screens and photographic paper.

(B) Flawless photographic technique is necessary to achieve the maximum image resolution possible with the equipment and materials being used.

(C) Software used to represent and print images produced by digital cameras do not have any significant effect on the image quality.

(D) The only factors important in determining the degree of detail reproduced in the final photographic image are the resolution of the camera’s sensor and the resolution of the electronic screen.

(E) The definition of the term “resolution” does not represent a significant determinant of the quality of photographic instruments and their results.

4. People are often told to eat a substantial breakfast because it reduces the amount of food consumed during the rest of the day. Yet, a recent study reported that for both obese and normal weight adults, eating a large, small, or no breakfast made no difference at all with regard to their non-breakfast calorie intake during the rest of the day.

Which of the following is most reasonably supported by the information above?

(A) Dieters who eat small breakfasts benefit from the same reduction of calories as those who eat no breakfast at all.

(B) Dieters who eat a large breakfast eat less food for the rest of the day but because they eat higher-calorie non-breakfast foods the calorie intake remains the same.

(C) Dieters who eat a small or no breakfast may actually reduce their calorie intake while those who eat a large breakfast probably increase their calorie intake.

(D) Whether dieters eat a large, small, or no breakfast makes no difference to their overall calorie intake.

(E) Dieters who eat no breakfast at all eat more during the rest of the day, but because they eat lower-calorie non-breakfast foods the calorie intake remains the same.

5. A group of 1,000 homeowners was randomly selected from three cities in the Midwest and asked the question “Do you plan to pay off your mortgage in full through regular payments or the sale of your home?” More than 92 percent answered, “Yes.” This shows that the overwhelming majority of homeowners want to pay off their mortgage, and that if the national mortgage default rate among homeowners is high, it cannot be due to a lack of desire on the part of homeowners.

The reasoning of the argument above is questionable because the argument

(A) attempts to make two conflicting conclusions using the results of one survey

(B) fails to justify its assumption that 92 percent is an overwhelming majority

(C) contradicts itself by admitting that there may be a high default rate among homeowners while claiming that most homeowners want to pay off their mortgages

(D) treats homeowners from three cities in the Midwest as if they are representative of homeowners nationwide

(E) overlooks the possibility that there may in fact not be a high default rate among homeowners

Questions 6–7

Professor Allyn:   The literature department’s undergraduate courses should cover only true literary works, and not such questionable material as political speeches.

Professor Raleigh:   Political speeches might or might not be true literary works, but they have a powerful and sometimes dangerous effect on society—largely because people cannot discern the actions implied by the words. The literature department’s courses give students the critical skills to analyze and understand texts. Therefore, it is the literature department’s responsibility to include the study of political speeches in its undergraduate courses.

6. Which one of the following principles most strongly supports Professor Raleigh’s argument?

(A) All undergraduate students should take at least one course that helps them develop critical skills.

(B) Political speeches should be discussed in a way that makes the implied actions clear.

(C) The literature department’s courses should enable students to analyze and understand all kinds of texts that can have a powerful and sometimes dangerous effect on society.

(D) Any professor teaching an undergraduate course in the literature department should be free to choose whatever materials are necessary to teach that course.

(E) All texts that are subtly constructed and capable of influencing people’s thoughts and actions should be considered a form of literature.

7. Which one of the following is an assumption on which Professor Raleigh’s argument depends?

(A) The literature department’s academic responsibility is not limited to teaching students how to analyze true literary works.

(B) Political speeches given at legitimate political rallies are not dangerous to society.

(C) The literature department does not teach students in their courses any other skills besides those needed to analyze and understand texts.

(D) Texts that are true literary works never are detrimental to society.

(E) Courses offered by the literature department cannot include both true literary works and material such as political speeches.

8. If a teacher gives a child only a few options for how he or she can spend time in class, the child is more likely to adhere to the teacher’s instructions than if the teacher gives the child many options.

Which one of the following most accurately expresses the principle illustrated above?

(A) Children dislike calculating the best of a variety of options unless they can see a clear difference among the benefits that would result from each option.

(B) The tendency children have to alter their behavior varies inversely with the number of alternative activities made available to them by an adult.

(C) Children are likely to ignore instructions from their teachers if they are confused about those instructions.

(D) To get good results, the clarity with which a teacher can instruct the children is of equal importance to the quality of the proscription for not following the teacher’s instructions.

(E) Most children are unlikely to follow their teacher’s instructions unless they can vividly imagine the consequences of not following the teacher’s instructions.

9. A fungus caused by spores called late blight can destroy whole crops of tomatoes and can reach epidemic levels in areas where tomatoes are grown. The spores that cause the fungus can be controlled with a synthetic fungicide, but the fungicide can pose health hazards to people living nearby. The fungicides are thus unsuitable for small farms, garden stores, and backyard gardens near populated areas. Fortunately, commercial tomato fields are in isolated locations where fungicides can be used safely. Therefore, most of the nation’s tomato crops are not seriously threatened by late blight.

Which one of the following is an assumption on which the argument depends?

(A) The fungus caused by late blight spores is the only disease that threatens tomatoes nationwide.

(B) It will eventually be possible to breed tomatoes that are resistant to late blight spores.

(C) The fungus caused by late blight spores spreads more slowly on commercial tomato fields than in small tomato farms, garden stores, and backyard gardens.

(D) Most of the tomato plants that have not been exposed to the late blight spores grow in small tomato farms, garden stores, and backyard gardens.

(E) Commercial fields produce most or all of the nation’s tomatoes.

10. Lawyer:   Our next objection will probably be overruled, because normally about half of all objections that the judge considers are overruled, and our last five objections have all been sustained.

The lawyer’s reasoning is flawed because it presumes, without giving warrant, that

(A) the judge is required to overrule at least half of all objections raised

(B) the likelihood that an objection will be overruled is influenced by the potential influence it may have over the expected verdict

(C) the last five objections having been sustained guarantees that the next five objections will be overruled

(D) having the last five objections sustained affects the likelihood that the next objection will be sustained

(E) the majority of the last five objections deserved to be sustained

11. Fifty years ago, the percentage of the nation’s tax revenues spent on the maintenance of the national parks was twice what it is today. Given that tax revenues have risen over the past 50 years, we can conclude that tax revenues have risen at a greater rate than the cost of maintaining the nation’s national parks.

Which one of the following is the assumption upon which the argument’s conclusion relies?

(A) In general, the national parks are in better condition today and less costly than they were 50 years ago.

(B) The national parks today have the same maintenance requirements as they did 50 years ago.

(C) Unlike the national parks, expenditures to maintain the federal buildings are the same percentage of tax revenues as they were 50 years ago.

(D) The amount spent per square mile of national park space is identical to the amount spent per square mile of national park space 50 years ago.

(E) The costs related to maintenance of other federal properties have risen faster than the price of maintaining the national parks over the past 50 years.

Questions 12–13

City mayor:   An independent group has released a new study that shows that more than 63 percent of our students graduate from high school. That is an increase of 15 percent from what the graduation rate was when I entered office. This proves that my education policies are a success and that we should continue implementing them.

Critic:   While I applaud your success at increasing the graduation rate I must point out that of those who graduated only 29 percent tested as being prepared for college and well-paying careers. That is less than half and proves that your policies are a complete failure.

12. The critic counters the mayor’s argument by

(A) questioning the mayor’s motives for reaching a certain conclusion

(B) asserting that measuring progress toward educational goals is difficult and therefore the statistics lack credibility

(C) attempting to show that the mayor’s description of the facts is misleading

(D) discrediting the mayor’s methods while applauding the goal

(E) disputing the accuracy of the figures cited by the mayor

13. Which one of the following could the mayor properly cite as indicating a flaw in the critic’s reasoning concerning the education study?

(A) The high school students that tested as being unprepared for college and well-paying careers are isolated to a particular region of the city.

(B) The high school students that tested as being prepared for college and well-paying careers did not, in fact, continue on to enter college or begin well-paying careers.

(C) The increase in graduation rate was due to the annexation of a school district with a high graduation rate among its high schools.

(D) The test for preparedness is a new test and does not have any previous results with which to compare the current results.

(E) Other cities in the same state have a lower rate of preparedness for college and well-paying jobs.

14. The retail price of bleached flour is considerably higher than that of unbleached flour. However, the process by which flour is bleached is fairly simple and not very costly. Therefore, the price difference cannot be accounted for by the greater cost of providing bleached flour to the consumer.

The argument relies on assuming which one of the following?

(A) Grocery stores do not expect that consumers are willing to pay more for bleached flour than for unbleached flour.

(B) There is little competition among companies that process bleached four.

(C) Price differences can generally be accounted for by such factors as supply and demand for the products, not by differences in production costs.

(D) The only factor relevant to the cost of providing bleached flour to the consumer is the cost of the bleaching process.

(E) Processing unbleached flour costs more than processing bleached flour.

Question 15–16

A newspaper article on lawsuits in the United States argued that they are on the decline as a method of resolving disputes. The article’s evidence was the decreasing number of court verdicts in civil cases, as if the only method of resolving civil cases is through a lengthy and expensive court hearing. Surely, in a modern legal system, the fact that a case reached a court verdict is a sign that the legal system failed to resolve the dispute by other means. The parties to a lawsuit have ways other than through the courts to resolve their differences, such as settlement through arbitration and mediation.

15. The argument criticizing the newspaper article is directed toward establishing which one of the following as its main conclusion?

(A) There is no reason to believe, on the basis of what the newspaper article said, that lawsuits as a method of resolving disputes are on the decline.

(B) Without the possibility of lengthy and expensive court hearings, professionals involved in arbitration and mediation would not be thriving.

(C) Because court hearings are lengthy and costly, other methods of resolving disputes such as arbitration and mediation are necessary.

(D) Lawsuits are unsuccessful if the only way of resolving the dispute is through a court hearing.

(E) Although lawsuits are a popular and effective method for citizens to gain remuneration for being wronged, that does not preclude them from using other methods to resolve their disputes.

16. The argument criticizing the newspaper article employs which one of the following strategies?

(A) arguing that the article’s conclusion is motivated by a desire to promote the reduction in number of lawsuits

(B) detailing historical changes that make the article’s analysis outdated

(C) pointing to the common interests between the lawyers and judges who manage court hearings, which the article ignores

(D) reinterpreting evidence that the article uses as indicating the negation of what the newspaper concludes

(E) questioning the accuracy of the statistical evidence that the article uses

17. President of company X:   Did the manager hire the best engineer to design the new product?

Vice president:   Yes.

President:   And the best production team?

Vice president:   Yes.

President:   In fact everyone he assigned to designing and manufacturing the product was the very best he could find?

Vice president:   That’s correct.

President:   So, your report deliberately misrepresented the manager’s performance when you claimed he never really wanted the new product to succeed.

Each of the following accurately describes a flaw in the president’s reasoning displayed above EXCEPT:

(A) It takes for granted that the product could fail only if the manager wanted it to fail.

(B) It ignores the possibility that the vice president failed to make the correct inferences from the facts known and therefore his report’s negative assessment was unintentional.

(C) It takes for granted that the manager was not forced to assign the people he did to design and manufacture the product.

(D) It ignores the possibility that the manager failed to allot enough time or resources to the production team.

(E) It ignores the possibility that the manager knew that the people assigned to the product would not work well together.

18. My brother likes spinach, but not cabbage, which he says is too bitter. So it is not true that whoever likes cabbage likes spinach.

The flawed reasoning in the argument above most closely resembles that in which one of the following?

(A) Sophia enjoys managing computer networks, but not computer programming, which she says is tedious. So it is not true that whoever enjoys managing computer networks enjoys computer programming.

(B) Although a man is more than seven feet tall, he is not considered a giant. Thus, it is not the case that all giants are more than seven feet tall.

(C) All minimalist music compositions were written after the year 1960. This music composition is minimalist, so it must be true that it was written after the year 1960.

(D) People who repair their own plumbing are do-it-yourself fanatics. My next-door neighbors are do-it-yourself fanatics, so it follows that they repair their own plumbing.

(E) This photographic print is not in color, but it is expensive. So it is not true that some color photographic prints are expensive.

19. Expert witness: We have tested the explosive used in the bank robbery 10 times under controlled circumstances. Each time we detonated 5 square inches of the explosive substance coated with a dye to measure the distance of the explosion. In all 10 cases, the dye covered an area much less than 20.7 square feet. In fact, the dye-covered area was always between 9.2 and 12.5 square feet. I conclude that 5 square inches of the explosive substance destroys an area much less than 20.7 square feet.

Which one of the following, if true, most undermines the value of the expert’s evidence as basis for the conclusion?

(A) Another explosive substance was substituted, and in otherwise identical circumstances, the dye covered between 21.1 and 23.1 square feet.

(B) Not all expert witnesses are the authorities in their fields that they claim to be.

(C) Expert witnesses are notoriously unreliable because they tend to adjust their evidence to support the prosecution’s case.

(D) On the eleventh detonation of the explosive substance the area covered was also less than 20.7 square feet—this time covering 20.1 feet.

(E) If similar results had been found after 100 test detonations of the explosive substance, the evidence would be stronger.

20. Historian:   The spread of access to the Internet allows more people to learn about injustices and, in the right circumstances, leads to an increased capacity to distinguish true reformers from mere opportunists. However, widespread access to the Internet invariably comes faster than the development of a generally enlightened education system. In the interim, the populace is vulnerable to clever charlatans calling for change. Consequently, some relatively reasonable regimes may be toppled by their own progressive policies to bring new technologies to the masses.

Which one of the following is an assumption on which the historian’s argument depends?

(A) A lack of enlightened education affects the ability to differentiate between legitimate and illegitimate calls for reform.

(B) Any reasonable regime that fails to provide a generally enlightened education system will be toppled by a clever charlatan.

(C) A charlatan can never enlist the public support necessary to topple an existing regime unless a generally enlightened education system is in place.

(D) Without access to the Internet there can be no general awareness of injustice in a society.

(E) Any generally enlightened education system will tend to preserve the authority of reasonable regimes.

21. Many sleep aid drugs increase appetite. While dieting can help counterbalance the increased weight gained from taking such sleep aids, some weight gain is unlikely to be preventable.

The information above most strongly supports which one of the following?

(A) At least some patients taking sleep aid drugs gain weight as a result of taking them.

(B) All patients taking sleep aid drugs should diet to maintain their weight.

(C) The weight gain experienced by patients taking sleep aid drugs should be attributed to lack of dieting.

(D) A physician should not prescribe any sleep aid drugs for a patient if that patient is overweight.

(E) People who are trying to lose weight should not ask their doctors for sleep aid drugs.

22. Intelligence is a limited resource, and education and training helps to use this resource efficiently. Since the most successful people do not differ greatly from each other in intelligence it follows that a requirement for a person to achieve success is a superior education.

Which one of the following most accurately expresses the conclusion of the argument?

(A) Education helps people use limited resources efficiently.

(B) No person can become successful without a superior education.

(C) Only a successful person has a superior education.

(D) The differences in intelligence between successful people are not great.

(E) Superior intelligence is a requirement for a person to be successful.

23. Attacks on your opponent’s character should be avoided before a football game. Such attacks have nothing to do with the opponent’s competitiveness; instead they attempt to question the opponent’s moral right to compete in the game at all.

Which one of the following principles, if valid, most helps to justify the reasoning above?

(A) Attacking the character of one’s opponent does nothing to preserve one’s moral right to play in further football games.

(B) Questions of character should be raised before a football game if they are relevant to the opponent’s ability to compete on the field.

(C) Attacks on the opponent’s character should not impress the spectators at the game.

(D) Attacks on an opponent’s character result from an inability to compete effectively with the opponent on the field.

(E) Behaviors that have nothing to do with the aspects of playing the game on the field should be avoided.

24. In order to hire a new employee a manager cannot post an opening without first getting approval from the manager’s superior. Unfortunately, the manager’s superior cannot approve the request if the manager does not already have approval from human resources. Any manager who wants to hire a new employee, therefore, must gain approval from human resources.

The argument above is most similar to which one of the following?

(A) Among the football players on the Chicago team any player who played against Dallas also played against New Orleans, and some players who played against New Orleans also played against Seattle. Therefore, some players on the Chicago team who played against Dallas have also played against Seattle.

(B) A doctor cannot prescribe riflinine for a patient without first prescribing cenoflan for that patient. Unfortunately, cenoflan makes most patients extremely nauseated or extremely hyperactive. It is likely, therefore, that a patient who has taken riflinine has felt extremely hyperactive.

(C) A martial arts student cannot achieve the brown belt without first achieving the purple belt. The purple belt is impossible to achieve unless the blue belt has already been achieved. Therefore, a martial arts student who has reached the brown belt must have previously achieved the blue belt.

(D) It is impossible to be a law-abiding citizen without knowing the society’s laws. A person who knows the laws of a society has learned them by means of exhaustive and difficult study or else by growing up among law-abiding citizens within that society. There are two major ways, therefore, for a person to become a law-abiding citizen.

(E) One cannot properly identify a bacterium without examining the flagellum that enables its movement. A powerful microscope can be used to examine the flagellum of a bacterium. A powerful microscope, therefore, is necessary for anyone wishing to identify a bacterium properly.

25. French film director Claude Morrel was accused by Russian film director Slovan Stipich of plagiarizing a movie he made that had been released 15 years before Morrel’s. The two movies are set in different periods and regions, but they contain enough plot similarities to make the resemblance unlikely to be coincidental. Morrel’s defense rests on his argument that plagiarism was impossible because Stipich’s movie was made in Russian, a language Morrel does not understand, and because the movie was never subtitled or reviewed in any other language.

The argument in Morrel’s defense depends on the assumption that

(A) there is a common myth between both cultures to which both directors referred to subconsciously in the movies in question

(B) Morrel is familiar with an old Cyrillic language that is extinct but related to the modern Russian language

(C) Morrel has never met Stipich

(D) Stipich’s movie did not become popular in Russian

(E) nobody related the plot of Stipich’s movie in detail to Morrel before Morrel produced his movie

STOP

IF YOU FINISH BEFORE TIME RUNS OUT, CHECK YOUR WORK ON THIS SECTION ONLY. DO NOT GO ON TO ANY OTHER TEST SECTION.

SECTION III

Time—35 minutes

24 Questions

Directions: Each passage in this section is followed by a group of questions to be answered on the basis of what is stated or implied in the passage. For some of the questions, more than one of the choices could conceivably answer the question. However, you are to choose the best answer; that is, the response that most accurately and completely answers the question, and blacken the corresponding space on your answer sheet.

PASSAGE A

Images

Images

PASSAGE B

Images

Images

1. Which of the following issues is central to both passages A and B?

(A) As globalization takes hold of nations across the world they are leaving behind their local and traditional legal systems to build more Western-style institutions.

(B) As globalization takes hold of nations across the world many newly independent nations are designing constitutions to better integrate with both the world at large and their own people’s interests.

(C) Some nations, as they transition to Western-style legal systems, have found that these systems occasionally conflict with traditional customs and legal structures.

(D) As nations transition to more Western-style legal systems the adjudication of legal cases has been taken over by the rule of law from traditional, more informal institutions.

(E) Nations, as they transition to more Western-style legal systems, have suffered a new type of imperialism that is more harmful than what they suffered over the last 100 years.

2. What is the main purpose of the second paragraph of passage A?

(A) exhibit how legal pluralistic arrangements have been highly successful in one particular area of the world

(B) describe the forms of legal pluralism that have existed through the years

(C) argue that legal pluralism is an unworkable solution for governments trying to transition to modernity

(D) describe the manner in which an abstract problem described elsewhere has actually arisen

(E) explain how legal pluralism has only led to disastrous results throughout history and into modernity

3. According to information in passage A, which of the following would best describe the attitude of newly independent governments towards traditional leaders?

(A) respectful tolerance

(B) disdainful accommodation

(C) angry resentment

(D) suspicious unease

(E) uncertain anxiousness

4. The author of passage B uses the example of Zimbabwe’s attempts to reform its water management system in order to

(A) describe the extent to which Zimbabwe has gone to bring its country into the family of modern nations

(B) contrast the local traditional method of dealing with administrative issues with that of the national government

(C) illustrate the range of difficulties experienced by Zimbabwe in its legally pluralistic society

(D) exemplify the conflict between foreign interests and the traditional institutions on a particular health-related issue

(E) introduce a circumstance when the government failed to work with the local tribal chiefs to modernize the nation’s infrastructure

5. Which of the following best describes the relationship between passage A and passage B?

(A) Passage A presents an alternative perspective on legal pluralism that passage B rejects.

(B) Passage A offers a resolution to the conflict inherent to legal pluralism and passage B questions the validity of that solution.

(C) Passage A discusses the origins of legal pluralism and passage B suggests that a global human rights conflict is attributable to the practice of legal pluralism

(D) Passage A discusses a tension inherent to legal pluralism and passage B presents a specific example that illustrates that tension.

(E) Passage A discusses the issue of globalization and human rights and passage B places this issue in its proper context.

Images

Images

Images

Images

6. Which one of the following best states the main idea of the passage?

(A) Critics have misinterpreted Faulkner’s novel as an allegory of the breakdown of the social fabric of southern society in the early twentieth century.

(B) Critics have come to interpret Faulkner’s novel as a reinterpretation of the southern gothic novel and a new representation of its complexities.

(C) Critics have discovered that while Faulkner’s novel explores aspects of southern social structures, it also explores the limitations of the novel as a storytelling device.

(D) Critics have failed to come to any definitive interpretation of Faulkner’s novel because the very nature of the novel is one that questions whether interpretation is even possible.

(E) Critics have come to realize that Faulkner worked collaboratively with fellow Hollywood screenwriters in writing his novel and thus call into question its authorship.

7. The passage suggests that the author would be most likely to agree with which one of the following statements about the contemporary critics of Faulkner’s Absalom, Absalom!?

(A) The critics fail to take into account the central story of Henry Sutpen and his rise to power, which aligns with Faulkner’s main concern, the breakdown of southern culture.

(B) The critics’ theories may not take into account the influences on Faulkner’s style, but the novel’s structure and its conclusion give some credence to them.

(C) The critics have become too focused on structuralist concepts that have led them on a tangent when it comes to addressing the dominant themes of Faulkner’s novel.

(D) The critics allowed the influence of Hollywood and detective novels on Faulkner’s writing to color this perspective and depart from accepted critical notions.

(E) The critics have done an excellent job of overthrowing all previously accepted theories concerning the novel and narrative theory.

8. The primary purpose of the second paragraph is to

(A) give evidence that casts some doubt on a critical theory

(B) offer biographical information that supports a critical theory

(C) describe the circumstances under which critics came to their conclusion

(D) detail the historical background that surrounded the creation of the novel

(E) clarify the underlying foundations of a particular critical theory

9. The passage implies which one of the following with regard to Faulkner’s character Quentin Compson in the novel?

(A) He was not in fact from the South even though he identifies himself as a Southerner.

(B) His character was inspired by Hollywood scriptwriters whom Faulkner encountered.

(C) He hates his southern heritage and his account of Sutpen’s history indicates as much.

(D) He appears in other novels and serves to express Faulkner’s own perspective.

(E) His character was derived through reading detective novels by Raymond Chandler.

10. The modern critics of Faulkner discussed in the passage would be most likely to agree with which of the following statements?

(A) Faulkner intended the Sutpen murder mysteries to represent the decay of the southern paternal system.

(B) It is the reader who must supply the evidence that is sorely lacking in the novel so that the mysteries at the heart of the Sutpen story can be solved.

(C) All novels are a collaboration between the novelist and the reader and only together can they unlock the mysteries put forward by the text.

(D) Despite Faulkner’s efforts to the contrary there are specifics revealed within the story that give incontrovertible proof of what really occurred during Sutpen’s life.

(E) The novel is an inherently misleading medium, given that the author’s voice is by its nature untrustworthy, demanding that the reader figure out which, if any, information in the novel is accurate.

11. According to the passage Faulkner’s writing of the novel was heavily influenced by all of the following EXCEPT:

(A) scriptwriting

(B) noir detective novels

(C) southern culture

(D) gothic story structure

(E) the author’s own struggles with a southern heritage

12. The author of the passage is primarily concerned with presenting which one of the following?

(A) historical background that may have affected the content of a novel

(B) an exploration of a critical theory as to why an author structured a novel as he did

(C) a conflict between literary critical approaches that has yet to be resolved

(D) an explanation of why an author included a mysterious chapter at the end of a novel

(E) a discussion of the influences that resulted in an author’s unique approach to writing a novel

Images

Images

Images

Images

Images

13. Which of the following best states the main idea of the passage?

(A) The theoretical nature of dark energy and the exorbitant cost of space exploration mean that scientists will never understand what is causing an ever-expanding universe.

(B) The disagreement between two opposing factions with irreconcilable theories on dark energy are hindering progress on understanding the ever-expanding universe.

(C) While the universe’s future, contraction or infinite expansion, remains unresolved, several theories concerning the force causing its expansion are fueling the current scientific discussion.

(D) Since Einstein’s theory of gravity has failed to deliver an explanation for dark energy, scientists have turned to the theory of quintessence because it lacks the burden of the cosmological constant.

(E) Until scientists obtain observational evidence through experiments in space they are left with the theoretical concept of dark energy, which has fueled several hypotheses as to how it works.

14. The author of the passage mentions in the first paragraph that it was known how much energy was necessary to cause the past and current rate of expansion of the universe in order to

(A) show that scientist do have some understanding of the larger universe even if they lack the specific understanding of what is causing it to expand

(B) show that scientists remain in a theoretical quandary with regard to the nature of dark energy

(C) prove that the universe is ever-expanding and will not contract as thought previously

(D) explain the high level of uncertainty with scientists’ theoretical models for explaining dark energy

(E) give credence to the theory that dark energy constitutes the vast majority of the makeup of the universe

15. It can be inferred from the passage that which one of the following is true of quintessence?

(A) It proves that Einstein’s theory of gravity is flawed and needs to be replaced with a new one.

(B) Observational evidence is unnecessary because its equations do not include the troublesome cosmological constant that burdened Einstein’s theory of gravity.

(C) Dark energy is not inherent in space itself and not consistently present throughout the universe.

(D) Quantum theorists do not have the same problem of finding too much energy as they did with Einstein’s equations.

(E) Dark energy replaces gravity and explains all forces, both attractive and expansionary.

16. The primary function of the passage as a whole is to

(A) offer theories of dark energy that are in conflict with each other

(B) offer theories of dark energy that have been challenged but remain under consideration due to lack of evidence

(C) offer theories of dark energy that have been discarded due to lack of evidence

(D) introduce evidence that supports one theory of dark energy over another

(E) make an argument in favor of expending public funds for experiments that will prove the nature of the expanding universe

17. According to the passage, all of the following are theoretically possible characteristics of dark energy EXCEPT:

(A) It is a naturally occurring phenomenon of space itself.

(B) It is increasing with the expansion of the universe.

(C) It supplants gravity as the dominant force in the universe.

(D) Evolving rivers of it flow through space.

(E) It will eventually give way to gravity and the universe will contract.

18. If the author of the passage were to add one more sentence at the end of the last paragraph, which of the following would it be?

(A) Until financial resources become available scientists should move on to more practical pursuits.

(B) For now the mysteries surrounding what constitutes most of the known universe will remain unsolved for at least awhile longer.

(C) It is obvious that such theoretical concepts are no longer of concern to the society at large and thus should be abandoned.

(D) Since neither Einstein’s theory of gravity nor the theory of quintessence have been vindicated, scientists must find a new and more viable theory to explain dark energy.

(E) It is impressive that concepts developed more than 100 years ago by Einstein and several hundreds of years ago by the Medieval scientists still persist today even if they are unsupported by any hard evidence.

Images

Images

Images

Images

19. Which one of the following titles most accurately describes the contents of the passage?

(A) The New Proposition: Privacy in the Defense of Property

(B) In Transition: How Privacy Moved from the Private to Public Consideration

(C) Creativity Under Defense: The Legal Profession Takes on Copyright Law at the Turn of the Century

(D) The Essay with Personality: Two Men Redefine a Nation’s Definition of Privacy Under the Law

(E) Privacy Gets Real: Warren and Brandeis Extend Privacy Protection to Thoughts, Sentiments, and Emotions

20. It can most reasonably be inferred from the passage that Brandeis and Warren’s approach to the right to privacy was

(A) based on their observations of serious deficiencies in the current law

(B) intended to protect their own property interests, specifically some portraits each of them owned

(C) revenge upon those who had slandered their acquaintances

(D) intended to neutralize actions directed at Warren and his acquaintances as objects of public gossip

(E) intended to protect some very talented and creative people among their relatives

21. The passage suggests that the other scholars referred to in the passage would be most likely to believe which one of the following statements?

(A) If person B steals a short story written by person A and publishes it word for word in a local pamphlet, A’s privacy has been infringed upon.

(B) If person B steals person A’s photograph from A’s home and publishes the photograph, A’s privacy has not been infringed upon.

(C) If person A writes a story that has not yet been published and person B reads it and later publishes his or her own version of the story, A’s privacy has been infringed upon.

(D) If person A tells person B a personal anecdote and person B publishes that anecdote in the local newspaper, person A’s privacy has been infringed upon.

(E) If person A gives a photograph to person B and person B, in turn, publishes that photograph in a local newspaper, person A’s privacy has been infringed upon.

22. According to the passage, Brandeis and Warren’s approach to personality was

(A) an evolution of earlier ideas

(B) a tweak to existing law that had larger ramifications

(C) a discounted side step from their central argument

(D) a revolutionary expansion of the existing law

(E) a redefinition, which was poorly understood by existing scholars, of an existing concept

23. The function of the second paragraph is to

(A) clarify elements within the essay that led to their conclusions

(B) explicate the structure of the essay

(C) explore the underlying cultural phenomena that influenced the writers

(D) give the reasons that they wrote the essay

(E) portray the writers and their personal reasons for writing the essay

24. The primary purpose of the passage is to

(A) discredit an archaic legal argument

(B) explore the current influence of a historic legal essay

(C) establish the historic importance of a landmark legal essay

(D) delineate between two different conceptions of the right to privacy

(E) discuss the circumstances and nature of a historic legal argument

STOP

IF YOU FINISH BEFORE TIME RUNS OUT, CHECK YOUR WORK ON THIS SECTION ONLY. DO NOT GO ON TO ANY OTHER TEST SECTION.

SECTION IV

Time—35 minutes

27 Questions

Directions: The questions in this section are based on the reasoning contained in brief statements or passages. For some questions, more than one of the choices could conceivably answer the question. However, you are to choose the best answer; that is, the response that most accurately and completely answers the question. You should not make assumptions that are by commonsense standards implausible, superfluous, or incompatible with the passage. After you have chosen the best answer, blacken the corresponding space on your answer sheet.

1. Arbus:   It has been argued that adding a 3-D effect to a movie degrades the integrity of the original 2-D version, and that nobody should be required to wear special glasses to experience a work of art. But nobody argues that we should not transform a theatrical movie into a home video that people can view at home because it erodes the value of the theatrical version, nor do they argue that people should not be required to use a video player to watch it. The home video version is a technologically different production that stands on it own. Judgments of it do not reflect on the original theatrical version. Similarly, a 3-D version of a movie is a distinct version from the original and should be judged on its own merit. It does not degrade the integrity of the original 2-D version.

Arbus’s argument uses which one of the following techniques of argumentation?

(A) It appeals to an inference from a general principle and a set of facts.

(B) It draws on popular opinion on the matter at issue.

(C) It proffers an example counter to a general principle.

(D) It invokes an analogy between similar cases.

(E) It distinguishes facts from value judgments.

2. During the meeting of the G-20 summit of major economic countries the member nations authorized a rescue package to aid one of its members in handling a debt crisis that threatened its economic future. Afterward, the parliament of one of the G-20 members passed a resolution condemning its own prime minister for promising to contribute funds to the rescue package. A parliamentary leader insisted that the overwhelming vote for the resolution did not imply the parliament’s opposition to the financial intervention; on the contrary, most members of parliament supported the G-20 action.

Which one of the following, if true, most helps to resolve the apparent discrepancy presented above?

(A) In the parliamentary leader’s nation, it is the constitutional prerogative of the parliament, not of the prime minister, to initiate financial aid to foreign entities.

(B) Members of the parliament traditionally are more closely attuned to public sentiment, especially with regard to the use of public money, than are prime ministers.

(C) The public would not support the financial rescue unless it was known that the parliament approved of the action.

(D) The G-20 nations cannot legally commit funds of a member nation to a financial intervention.

(E) The treasury would be responsible for providing the funding vouchers necessary to move the money across borders for such rescue packages.

3. The game Complicus requires a great deal of manual dexterity. Laurie is a highly capable auto mechanic. Therefore, Laurie would make an excellent Complicus player.

The flawed pattern of reasoning in the argument above is most similar to that in which one of the following?

(A) Any good cyclist can learn to in-line skate eventually. Katherine is a champion cyclist. Therefore, Katherine could learn to in-line skate in a day or two.

(B) The role of Ebenezer Scrooge in community productions is often played by an experienced actor. Irving has played Ebenezer Scrooge in the community production for several years. Therefore, Irving must be an experienced actor.

(C) People who work with their hands for a living invariably enjoy do-it-yourself projects. Larry has been a successful construction manager for many years. Therefore, Larry enjoys do-it-yourself projects.

(D) People with long arms make good volleyball players. Everyone in Jackie’s family has long arms. Therefore, Jackie would make a good volleyball player.

(E) All horseracing jockeys have excellent balance. Ricardo is a champion cyclist. Therefore, Ricardo would make a good horseracing jockey.

4. Psychologist:   Doctors should stop refusing to prescribe drugs to help people with depression. Most cases of depression that psychiatrists treat are known to be caused by chemical imbalances. This suggests that people suffering from depression do not need months or even years of therapy, but rather need the right drugs to alter their biochemistry and alleviate what is causing their depression.

Each of the following describes a flaw in the psychologist’s reasoning EXCEPT:

(A) It neglects the possibility that for some people the available drugs are completely ineffective at treating their types of chemical imbalances.

(B) It presumes, without providing any evidence, that depression contributes to a chemical imbalance.

(C) It fails to consider the possibility that for some people therapy is the only treatment known to be effective for cases of depression not caused by a chemical imbalance.

(D) It presumes, without providing justification, that the cases of chemical imbalance psychiatrists treat are representative of all cases of chemical imbalance.

(E) It overlooks the possibility that therapy could help depressed people cope with their chemical imbalance.

5. An antidote for whooping cough has been developed, but researchers warn that its widespread use could be dangerous, despite the fact that this drug has no serious side effects and is currently very effective at limiting the duration and severity of whooping cough.

Which one of the following, if true, helps most to reconcile the apparent discrepancy indicated above?

(A) The drug can be fatal when misused, such as taking larger-than-prescribed doses.

(B) The drug does not prevent the spread of whooping cough from one person to another, even when the drug eventually cures the disease in the first person.

(C) The drug must be administered several times a day, so patient compliance is likely to be low.

(D) Use of the drug contributes to the development of deadlier strains of whooping cough that are resistant to the drug.

(E) The drug is very expensive and making it widely available would be infeasible.

6. Juan:   It is wrong to think that the same managerial style should be used with all employees. For many employees their work experience has been more team oriented than others and they would therefore function better on group, rather than solo, projects. An employee’s accustomed style of work environment should always dictate how they are managed.

Dorothy:   No, not always. Flexibility in the workplace, being able to work either on one’s own or on a team, is invaluable to a company and its ability to function in the marketplace.

The conversation lends the most support to the claim that Juan and Dorothy disagree on which one of the following?

(A) It is sometimes desirable to tailor managerial styles to the way the employee functions best.

(B) The main purpose of management is to train employees to be flexible within the work environment of the company.

(C) All employees should learn to adapt to various managerial styles.

(D) Many employees would work better on a team rather than solo projects.

(E) All employees work better when assigned solo projects.

7. Political consultant:   Most people do not have every word they utter judged for consistency, but every speech given by a politician is closely analyzed. That is why politicians give such consistent speeches, although such speeches are often not explicit on details.

The political consultant’s statements, if true, most strongly support which one of the following?

(A) Only politicians have every speech they give judged for consistency.

(B) No other people give more consistent speeches than do politicians.

(C) By having every word closely analyzed, some people are caused to be very consistent with their statements.

(D) Some people are highly consistent with their statements in spite of the fact that not every word they utter is judged for consistency.

(E) A speech given by a politician is generally evaluated more strictly than the majority of things said by other people.

8. Media consultant:   Electronic media are bound to bring an end to the traditional news organizations in our society. This is because the emergence of the traditional news organization, characterized by a group of journalists managed by an erstwhile news editor, was facilitated by the low cost and ease of publishing and distributing a newspaper. Currently, however, newspapers are being overtaken by electronic media. So, it is inevitable that the traditional news organization will not survive in our society.

The reasoning in the consultant’s argument is flawed because it

(A) relies inappropriately on an expert’s opinion

(B) confuses the value of an institution with the method by which it operates

(C) presupposes as a premise what it is trying to prove

(D) presupposes that just because something might happen it will happen

(E) mistakes something that enables an institution to arise for something necessary for the survival of the institution

Questions 9–10

David:   The effort of advanced nations to create new biofuels has increased demand for the world’s crops and diverted them from being used as food. As a result, prices for food have risen, increasing world hunger and political instability across numerous developing world nations. Advanced nations should scale back their efforts to produce more environmentally friendly fuels.

Lucie:   There are many other factors that could be responsible for driving up the prices of food. Last year severe weather destroyed crops of wheat in Russia and China. Also, a mealy bug infestation decreased Thailand’s output of cassava.

9. Which one of the following most accurately describes Lucie’s criticism of David’s explanation?

(A) It points out that David’s explanation is based on two hypotheses that contradict each other.

(B) It cites an analogous case in which David’s explanation clearly cannot hold.

(C) It offers an alternative explanation that is equally supported by the evidence that David cites.

(D) It refers to sources of additional data that cannot easily be reconciled with the facts David cites.

(E) It cites facts that suggest David’s argument overlooks alternative explanations.

10. Which one of the following, if true, could be used by David to counter Lucie’s rejection of his argument?

(A) Russia, China, and Thailand are not the only countries supplying crops for use in developing biofuels.

(B) Wheat and cassava are minor crops in the development of biofuels.

(C) The amount of crops affected by severe weather and pest infestation has remained unchanged for the last five years.

(D) Infestations by the mealy bug can be easily managed by introducing farmers to new pesticide technologies.

(E) Political instability is directly attributable to food riots over price increases.

11. A safety report indicates that, on average, automobile accidents decline by about 9 percent in those areas in which the city has reduced the number of traffic signs posted. In a certain city, the city reduced its number of traffic signs by 50 percent and over a three-year period the number of automobile accidents remained the same.

Which one of the following, if true, does NOT help resolve the apparent discrepancy between the safety report and the city’s public safety records?

(A) In the last three years, most of the automobile accidents occurred due to a lack of attention to traffic signs.

(B) Bureaucratic errors left many of the more accident-prone areas of the city still with the same number of traffic signs as before the reduction.

(C) The city now includes accidents involving pedestrians in its yearly total of automobile accidents, whereas three years ago it did not.

(D) Three years ago speed limits in the city were increased by as much as 10 kph (6 mph).

(E) In the time since the city reduced its number of traffic signs the city has experienced a higher than average increase in automobile traffic.

12. Jack said he was not going to include members of the marketing department in the project-planning meeting. However, among the items each attendee left the meeting with was a sales report that Jack felt was important. Since members of the marketing department had promised to produce just such a sales report and intended to give it to Jack at the meeting, at least some members of the marketing department must have been at the project-planning meeting.

A reasoning error in the argument is that the argument

(A) fails to establish that something true of some people is true of only those people

(B) uses a term that is innately evaluative as though that term was purely descriptive

(C) treats the evidence of someone’s presence at a given event as an assurance that that person had a legitimate reason to be at that event

(D) overlooks the possibility that a person’s interest in one kind of thing is compatible with that person’s interest in another kind of thing

(E) disregards the possibility that a change of mind might be warranted by a change in circumstances

13. If you have no skill with tools at all you will not be able to repair the water heater. And if you are not able to repair the water heater you will not be able to perform the duties of a maintenance engineer.

If the statements above are true, which one of the following must be true?

(A) If you are not able to perform the duties of a maintenance engineer, you are not able to repair a water heater.

(B) If you are able to perform the duties of a maintenance engineer then you have at least some skill with tools.

(C) If you have some skill with tools, you will be able to perform the duties of a maintenance engineer.

(D) If you are able to repair a water heater, you will probably be able to perform the duties of a maintenance engineer.

(E) If you are not able to perform the duties of a maintenance engineer you have no skill with tools.

14. Art critic:   The meaning of a work of art is ever shifting, not fixed, and therefore it may attract many equally valid interpretations. Interpretations essentially involve imposing meaning on a work of art, rather than discovering meaning in it, so interpretations need not consider the intentions of the artist. Thus, any interpretation of a work of art reveals more about the critic than about the artist.

Which one of the following is an assumption required by the art critic’s argument?

(A) In order to truly understand a work of art one must know the artist’s history.

(B) A critic of a particular work of art can never know the true intentions of the creator of that work of art.

(C) An artist’s intentions are relevant to a valid interpretation of the artist’s work.

(D) A meaning imposed on a work of art reflects facts about the interpreter.

(E) There are no criteria by which to distinguish the validity of different interpretations of works of art.

15. The studies showing that the replacement of an older power plant with a modern one decreases the incidence of major illnesses do not distinguish between a conventional or nuclear power plant; their survey included at least some areas powered by a nuclear power plant. The studies may also be taken as showing, therefore, that there is no increased health risk associated with living next to a nuclear power plant.

The pattern of flawed reasoning in which one of the following is most similar to the pattern of flawed reasoning in the argument above?

(A) Research has shown that it takes more energy to produce a paper bag than a plastic bag, but after each is used and deposited as waste in a landfill a plastic bag lasts longer and takes up more space than a paper bag. There is, therefore, no more environmental harm from a plastic bag than from a paper bag.

(B) Research shows that there is no greater long-term health benefit connected to taking vitamin supplements than with a moderate increase in the intake of fruits and vegetables. Clearly, then, there is no long-term health risk connected to the failure to take vitamin supplements, so long as enough fruits and vegetables are consumed.

(C) Research has shown that young people who drive a car with an accompanying adult and receive one full year of intensive driving instruction are less likely to become involved in accidents than those who simply pass a driving test and start driving on their own. This shows that adults are inherently more responsible drivers than young people.

(D) Research shows that the incidence of cancer is decreased by eating fruits and vegetables. The fact that this benefit exists regardless of whether they are grown conventionally or organically shows that there is no increased cancer risk to eating fruits and vegetables containing pesticide residues.

(E) Research has shown that there is no long-term health risk connected to a diet consisting largely of foods high in saturated fat and cholesterol if such a diet is consumed by someone who is genetically predisposed to process such a diet. Therefore, the health of a person’s parents is more important than diet.

Questions 16–17

Mr. Anderson:   I am upset that my daughter’s entire soccer team has been suspended for two games because some of her team members were taunting members of the opposing team. She was not taunting them, and it was clear to everyone who the culprits were.

League director:   I’m sorry you are upset, Mr. Anderson, but your daughter’s situation is like being stuck at the airport because of a delayed flight. People who aren’t involved in causing the delay nevertheless have to suffer by waiting there.

16. If the league director is speaking sincerely, then it can be inferred from what the league director says that the league director believes that

(A) Mr. Anderson’s daughter might not have taunted members of the opposing team

(B) a flight delay is generally caused by weather or inefficiencies in the air traffic control system

(C) Mr. Anderson’s daughter knows who it was that taunted members of the opposing team

(D) being suspended from two games will deter future unsportsmanlike behavior

(E) many team members were taunting members of the opposing team

17. The league director’s response to Mr. Anderson’s complaint is most vulnerable to criticism on which one of the following grounds?

(A) It attempts to confuse the point at issue by introducing irrelevant facts about the incident.

(B) It makes a generalization about all the members of the team, which is not justified by the facts.

(C) It assumes that Mr. Anderson’s daughter is guilty when there is evidence to the contrary that the director has disregarded.

(D) It suggests that taunting members of the opposing team produces as much inconvenience as does being caught at the airport due to a flight delay.

(E) It does not acknowledge the fact that waiting at the airport due to a flight delay is unavoidable while the mass punishment was avoidable.

18. Editorialist:   The frontal lobe of a teenager is not as well developed as an adult’s and the frontal lobe is primarily responsible for recognizing good from bad actions and their consequences. This means that teenagers do not have the ability to make good decisions while driving an automobile. Therefore, teenagers should have additional restrictions put on their driver’s licenses.

If the statements above are true, which one of the following most weakens the argument that teenagers do not have the ability to make good decisions while driving an automobile?

(A) Studies have shown that a vast majority of automobile accidents are attributable to slow response time to adverse stimuli, which is not controlled by the frontal lobe.

(B) Brain research has shown that other lobes of the brain contribute to decision-making functions in the brains of both teenagers and adults.

(C) Studies have shown that teenagers who have been in automobile accidents have frontal lobes of a similar size to those in adult brains.

(D) It has been proven that the development of the parietal lobe of a teenager’s brain is equivalent to that in the brain of an adult.

(E) Adults with brain injuries to their frontal lobe are involved in more automobile accidents than adults with healthy brains.

19. Due to wider commercial availability of electronic books, sales of printed books have dropped significantly.

Which one of the following conforms most closely to the principle illustrated above?

(A) Because of the wide variety of high-quality home video recorders, sales of high-quality televisions have improved.

(B) Because a new brand of chewing gum entered the market, consumers reduced their consumption of an established brand of chewing gum.

(C) Because neither of the two most popular spreadsheet programs has all of the features consumers want, neither has been able to dominate the market.

(D) Because a child was forbidden to watch television until the child completed homework, that child avoided dawdling and focused on homework.

(E) Because of the rising cost of union labor, manufacturers began to make more extensive use of robots in the manufacturing process.

Questions 20–21

George:   It was wrong of Kristen to tell our boss that the reason the project will not succeed is that our company does not have the talent or resources to successfully execute all the steps necessary to make the project come to fruition. Saying such falsehoods can never be other than morally wrong and we do have the talent and the resources—Kristen just does not believe in the project and did not want to do the work.

20. The main conclusion drawn in George’s argument is that

(A) it is always wrong not to tell the truth

(B) the real reason Kristen did not support the project is that she does not believe in it and does not want to do the work

(C) it was wrong of Kristen to tell her boss that the project would not succeed because she believes the organization cannot execute all the steps necessary to make the project a success

(D) it is wrong to avoid expressing one’s own opinion by blaming the failure on deficiencies within the organization

(E) Kristen did not tell her boss the truth

21. The justification George offers for his judgment of Kristen’s behavior is most vulnerable to criticism on the grounds that the justification

(A) relies on an illegitimate appeal to pity to obscure the fact that the conclusion does not logically follow from the premises advanced

(B) attempts to justify a judgment about a particular case by citing a general principle that stands in far greater need of support than does the particular judgment

(C) judges behavior that is outside an individual’s control according to moral standards that can properly be applied only to behavior that is within such control

(D) ignores an important moral distinction between saying something that is false and failing to say something that one knows is true

(E) confuses having identified the cause of a given effect with having eliminated the possibility of there being any other causes of that effect

22. Research indicates that members of the police force generally were raised in economically disadvantaged households. For it was discovered that, overall, police officers grew up in communities with average household incomes that were lower than the average household income for the nation as a whole.

The reasoning in the argument is flawed because the argument

(A) does not take into account the fact that members of the police force generally have lower salaries than their counterparts in the private sector

(B) fails to take into account the fact that many police officers live in high-density urban communities, which generally have low average household incomes

(C) fails to note there are some communities with low average household incomes in which no members of the police force grew up

(D) presumes without justification that members of the police force generally were raised in households with incomes that are average or below average for their communities

(E) inappropriately assumes a correlation between household income and economic advantage

23. Not surprisingly, there are no astronauts under the age of twenty-one. And, as is well known, no one under the age of twenty-one can purchase alcohol. Finally, some scientists are astronauts, some purchase alcohol, and some are under twenty-one.

If the statements above are true, then on the basis of them which one of the following must also be true?

(A) Some astronauts neither purchase alcohol nor are scientists.

(B) Some scientists are neither astronauts nor purchase alcohol.

(C) Some people who purchase alcohol are not astronauts.

(D) All scientists either are astronauts, purchase alcohol, or are under twenty-one.

(E) No astronauts are twenty-one years old.

24. For years, invasive plant species like kudzu have smothered broad swaths of the local forests, taken over our wetlands, and clogged our waterways. Invasive plants have proven to be a potent threat to biodiversity. However, scientific studies show that invasive plant species are rarely the cause of native species’ extinctions.

Which of the following, if true, most helps to resolve the apparent discrepancy in the information above?

(A) Invasive plants are not considered a threat to human health and therefore biodiversity has less importance.

(B) Kudzu’s growth crowds out other invasive plants that could be harmful to biodiversity.

(C) While kudzu and other invasive plants can adversely affect biodiversity in a particular locality, they have less affect on a larger geographic scale, and most species range over such large geographic scales.

(D) The biodiversity of forests, wetlands, and waterways was decreasing even before being threatened by kudzu and other invasive plant species.

(E) Scientific studies attribute extinctions to industrial waste, pollution from automobiles, and other harmful human activities.

25. I have read her paper arguing for a new, more rigorous approach to analyzing literature and M frequently ascribes bad faith to scholars who disagree with her. It is troubling that M asserts that these scholars’ opinions are colored by laziness and loyalty to outdated schools of thought. Add to this that M has often shown herself to be arrogant, overly ambitious, and sometimes plain nasty, and it becomes clear that M’s paper does not merit attention from serious scholars.

The author of the above scholarly review commits which one of the following reasoning errors?

(A) dismissing an approach to literary criticism by giving a biased account of it

(B) failing to distinguish between the criteria of being true and of being sufficiently interesting to merit attention

(C) using an attack on the moral qualities of the author of the paper as evidence that the paper is not worthy of scholarly discussion of its truth

(D) presenting as facts several assertions about the paper under review that are based only on strong conviction and would be impossible for others to verify

(E) taking it for granted that a scholar is unlikely to do the work necessary and question an established school of thought

26. Regulatory regimens are created to institute fairness in the delivery of government services. Thus, despite growing dissatisfaction with complex regulatory systems, it is unlikely that regulations will be simplified.

The claim that regulatory regimens are created to institute fairness in the delivery of government services plays which one of the following roles in the argument?

(A) It is used to weaken the claim that regulations should be simplified.

(B) It is a conclusion for which the claim that regulations are unlikely to be simplified is offered as support.

(C) It is cited as evidence that regulatory systems are becoming more and more complex.

(D) It is a conclusion for which the only support offered is the claim that dissatisfaction with complex regulatory systems is growing.

(E) It is a premise offered in support of the claim that it is unlikely that regulations will be simplified.

27. Presidents cannot achieve greatness as long as they remain in the capital city. Their abilities to listen, analyze, and negotiate, which government functions hone, are useful to a leader, but an understanding of the citizen’s everyday experiences and frustrations can be obtained only by the immersion in communities around the country that is precluded by being a government functionary.

Which one of the following is an assumption on which the argument depends?

(A) Presidents cannot achieve greatness without an intuitive grasp of a citizen’s everyday experiences and frustrations.

(B) Participation in communities, interspersed with impartial observation of everyday experiences and frustrations, makes presidents great.

(C) No great president lacks the power to listen, analyze, and negotiate.

(D) Knowledge of the citizen’s everyday experiences and frustrations cannot be acquired by merely listening, analyzing, and negotiating in life.

(E) Presidents require some impartiality to get an intuitive grasp of a citizen’s everyday experiences and frustrations.

STOP

IF YOU FINISH BEFORE TIME RUNS OUT, CHECK YOUR WORK ON THIS SECTION ONLY. DO NOT WORK ON ANY OTHER TEST SECTION.

LSAT Practice Test 2 Answer Key

Section I

1. A

2. C

3. E

4. D

5. B

6. C

7. E

8. B

9. C

10. B

11. E

12. C

13. D

14. A

15. B

16. E

17. A

18. C

19. C

20. C

21. A

22. B

23. D

24. E

25. B

Section II

1. D

2. A

3. A

4. C

5. D

6. C

7. A

8. B

9. E

10. D

11. B

12. C

13. D

14. D

15. A

16. D

17. A

18. B

19. D

20. A

21. A

22. B

23. E

24. C

25. E

Section III

1. C

2. D

3. A

4. C

5. D

6. C

7. B

8. B

9. D

10. E

11. D

12. B

13. E

14. A

15. C

16. B

17. E

18. B

19. D

20. D

21. A

22. D

23. C

24. E

Section IV

1. D

2. A

3. E

4. B

5. D

6. C

7. C

8. E

9. E

10. C

11. A

12. A

13. B

14. D

15. D

16. A

17. E

18. A

19. B

20. C

21. B

22. D

23. B

24. C

25. C

26. E

27. A

Calculate Your Score

Complete the following table.

Your Raw Score

Images

Your Approximate Scaled Score

It is impossible to say with complete precision what raw score will translate to what scaled score on future LSATs, but here is a rough estimation.

Images

LSAT Practice Test 2 Answers and Explanations

SECTION I

Questions 1–5

As with all logic games, you follow the six-step process.

STEP 1:  Identify the game type.

This is a grouping game. You know it is a grouping game because the description says you must “choose” four scientists for a project. It also says the scientists must be “selected” according to certain rules that limit the composition of the group. Furthermore, the game gives you two qualifications that exclude or include the candidate from the project: publication and specialization.

STEP 2:  Begin your diagram.

You can create your diagram as a grid with four slots. You can abbreviate the scientists’ names to their initials. There are two ways you can handle the placement of the variables: you can list them outside the grid and then place them into the grid as they are assigned, or you can put them into the grid and select those who qualify or eliminate those who do not qualify when handling each question. Because it simplifies things visually, let’s put them in the grid as follows:

Images

STEP 3:  Symbolize the clues.

Clue 1: Exactly two published scientists and two unpublished scientists are selected.

You do not need to create symbolizations of this clue. You can simply put the number 2 below each column to show that the elements in each column must add up to 2.

Clue 2: Exactly two microbiologists and two immunologists are selected.

You do not need to create a symbolization of this clue either. As with the previous clue, you can put the number 2 to the right of each row to show that each row must add up to 2.

Clue 3: Either Klipp or Gross or both are selected.

Essentially, this clue is saying that there is no situation where neither K nor G is included in the group. If K is not in the group, then G must be in the group. If G is not in the group, then K must be in the group. Finally, both K and G can be in the group, but it cannot be the case that neither is in the group. You symbolize this clue as follows:

K / G / K & G

Logically, the above can be reduced to K/G. The logical disjunction “or” does not exclude that both can be in the group.

STEP 4:  Double-check your symbolizations.

To double-check your symbolizations, you translate your symbolized clues back into normal English and see whether they match the original language of each clue. In this case you have only one clue that has been symbolized. Once you have verified that it works, you can add the symbolizations to your diagram. Your diagram should look as follows:

Images

STEP 5:  Make deductions.

1. Can’t-be-first-or-last deductions

No deductions of this type are possible in this game.

2. Repeated-element deductions

No deductions of this type are possible in this game.

3. Down-to-two deductions

No deductions of this type are possible in this game.

4. Block-splitting deductions

No deductions of this type are possible in this game.

STEP 6:  Answer the questions in the smartest order.

As you take the test, answer the questions in this order:

1. Answer the Complete and Accurate List question.

2. Answer questions that give you more information to work with.

3. Answer the remaining questions.

Question 1 is a Complete and Accurate List question and should be answered first. Questions that give you more information include the following:

Question 2 (“Bosch and Klipp are selected for the research project . . .”)

Question 3 (“Bosch and Cristof are selected for the research project . . .”)

Question 4 (“Hoff and Mann are selected for the research project . . .”)

Question 5 (“N is selected for the research project . . .”)

There are no remaining questions.

THE ANSWERS

1. Answer: A

The question asks you to identify the answer that gives an acceptable selection of scientists for the project. This means that all but one of the groupings is unacceptable. You can quickly eliminate answer C because it does not include K or G, which is inconsistent with clue 3. You can look at each of the remaining scenarios using your diagram to determine whether it is acceptable or not. Answer A includes G and it gives you two unpublished, two published, two microbiologist, and two immunologist scientists. This is an acceptable selection and must be your answer. Let’s look at the remaining options for learning purposes. Answer B has three immunologists and only one microbiologist, so it is inconsistent with clue 2. Answer D has three microbiologists and only one immunologist, so it is inconsistent with clue 2 as well. Answer E has three microbiologists and only one immunologist, which is also inconsistent with clue 2. It also has three unpublished and only one published scientist, which is inconsistent with clue 1. The correct choice is answer A.

Images

2. Answer: C

The question tells you that both B and K are selected and then asks you to determine which answer gives you the two other scientists who must be selected with them. You use your diagram to determine the correct answer. Because B and K are both microbiologists, you determine that the other two must be immunologists in order to be consistent with clue 2. Also B is published while K is unpublished. Therefore, the other two must be split the same way, one published and one unpublished, so that the group is consistent with clue 1. The question already requires the selection of K, so you do not need to be concerned with abiding by clue 3. Restating your requirements, you need two immunologists, one who is published and one who is unpublished. This is exactly what answer C gives you, so the correct choice is answer C.

Images

3. Answer: E

The question tells you that B and C are selected and asks you to choose from among the possible answers the one scientist who must be selected. Of the two remaining slots, G or K must be one of them, but you cannot select G because that would put three published scientists into the group, which is inconsistent with clue 1. Therefore, the third slot must be taken by K, but K is not one of your possible answers, so you must determine which scientist occupies the fourth slot. Looking at the diagram, you notice that you need an unpublished immunologist in order to satisfy both clues 1 and 2. The only scientist that satisfies that requirement is J. The correct choice is answer E.

Images

4. Answer: D

The question tells you that H and M are selected for the group and asks you to select from among the possible answers the one scientist who could be, but need not be, selected. This means that all but one of the options either must be selected or cannot possibly be selected. The answer will be the one scientist whom you have the flexibility to include or not include. You use your diagram to evaluate the scenario. Clue 3 requires that you select either K or G for the group, but you cannot select K, because this would give you three microbiologists and three unpublished scientists, which is inconsistent with clues 1 and 2. Therefore, you can eliminate answer A as a possible answer. Because you cannot select K, you must select G for the group. Therefore, you can eliminate answer E as a possible answer. Answer B is impossible because selecting B would give you three microbiologists. Answer C is impossible because selecting J would give you three unpublished scientists. You are left with answer D. Selecting scientist C is consistent with the clues and gives you an acceptable group, but you do not necessarily have to select C. You could select E as well and it is this flexibility that leads you to select answer D. The correct choice is answer D.

Images

5. Answer: B

The question tells you that J is selected for the group and asks you to determine which of the answers gives you the scientist that must be selected as well. You use your diagram to analyze the scenario. J is the only unpublished immunologist, which means that in order for there to be two unpublished scientists, you must select only one from the unpublished microbiologist group. Also, in order for the group to have two immunologists, you must select only one from the published immunologist group. This leaves only one slot, and it must come from the published microbiologist group. Since B is the only published microbiologist, you must select B to the group. The correct choice is answer B.

Images

Questions 6–12

As with all logic games, you follow the six-step process.

STEP 1:  Identify the game type.

Though it is worded in a manner to suggest there is an ordering process (a tour usually follows along a particular path), this is a grouping game. The stops along the route are being assigned to “feature” groups—scenic views, historic landmarks, and restroom facilities. The tour operator has already chosen the six stops along the tour, so you are not scheduling the tour, but you are told that each stop has at least one of the features, so you know that you are assigning features to stops.

STEP 2:  Begin your diagram.

The game is to assign stops to features. You could work this game the other way around, assigning features to stops, but working with three pools rather than six will make your task simpler. You can diagram this game using a grid with the features abbreviated along the left side as SV, HL, and RF. The stops are along the top, and you will mark off the features with an X as you attempt to solve the problems given by the questions.

Images

STEP 3:  Symbolize the clues.

Clue 1: K has scenic views and restroom facilities.

This clue does not require symbolization. You can mark off the SV and RF boxes for K.

Clue 2: L has scenic views and a historic landmark.

Same as clue 1, this clue does not require symbolization. You can simply mark off the SV and HL boxes for L.

Clue 3: L and N have no features in common.

This says that L and N cannot have the same features. Therefore, it can be symbolized as follows:

L ≠ N

Clue 4: M has more features than L.

For your purposes, this is saying that M will have more feature boxes marked off than L. In the simplest terms possible you can symbolize this as follows:

M > L

Clue 5: K and P have exactly one feature in common.

You can use your box notations to symbolize this one as follows:

Images

Clue 6: J has fewer features than P.

This is similar to clue 4 and can be symbolized as follows:

J < P

STEP 4:  Double-check your symbolizations.

To double-check your symbolizations, you translate your symbolized clues back into normal English and see whether they match the original language of each clue. Once you have verified that your symbolizations work, you can add them to your diagram. Your diagram should look as follows:

Images

STEP 5:  Make deductions.

Finally, before you tackle the questions, you see if you can make any deductions based on the setup of the game and the clues. You go through your types of deductions.

1. Can’t-be-first-or-last deductions

No deductions of this type are possible in this game.

2. Repeated-element deductions

Stop K appears in clues 1 and 5. Clue 1 says that K has SV and RF. Clue 5 says that K and P have exactly one feature in common. Therefore, at least one of P’s features is either SV or RF, but it cannot have both. The next repeated-element deduction allows you to go further toward determining the feature set for P.

Stop P appears in clues 5 and 6. Clue 5 says that K and P have exactly one feature in common. You have already determined that P must have either SV or RF but not both in its feature set. Therefore, P cannot have all three features. At most, it can have two. Clue 6 says that J has fewer features than P. Since every stop must have at least one feature, then J must have one and P must have two, which means that P has HL as a feature. You write a number 2 below P’s column to indicate that it can have only one more feature (either SV or RF, of course). You write a number 1 below J’s column to show that it can have only one feature.

Stop L appears in clues 2, 3, and 4. According to clue 2, L has SV and HL. According to clue 3, L and N cannot have the same features. Therefore, N cannot have SV or HL. Since it must have at least one feature, it must have RF, the only remaining feature, so you will mark off N as having RF and only that feature on your grid; you can black out SV and HL. Clue 4 says that M has more features than L. Clue 2 tells you that L has two features. Therefore M must have all three features in order to have more features than L. You can mark off all three features for M in your grid.

3. Down-to-two deductions

No deductions of this type are possible in this game.

4. Block-splitting deductions

No deductions of this type are possible in this game.

Here is your new diagram with all your clues and deductions added to it:

Images

STEP 6:  Answer the questions in the smartest order.

As you take the test, answer the questions in the following order:

1. Answer the Complete and Accurate List question.

2. Answer questions that give you more information to work with.

3. Answer the remaining questions.

There are no Complete and Accurate List questions, so you must move on to questions that give more information. Those include:

Question 8 (“all the stops that have historic landmarks also have scenic views . . .”)

Question 9 (“P has no features in common with J but has at least one feature in common with every other stop . . .”)

Question 10 (“no two stops have exactly the same features as one another . . .”)

Question 11 (“exactly four of the six stops have historic landmarks, and exactly four of the six locations have scenic views . . .”)

Question 12 (“the condition requiring that M has more features than L is replaced by a new condition requiring that M and L have exactly two features in common . . .”)

The remaining questions are 6 and 7 and can be answered in that order.

THE ANSWERS

6. Answer: C

The question asks how many stops you can determine the features for without any further information. Through your deductions, you determined four of the stops, leaving J and P as uncertain. Therefore, the correct choice is answer C.

7. Answer: E

The question asks which of the answers must be false. This means that all but one of the answers could possibly be true. If J has RF and P has SV, then it is possible that four of the six stops have RF, so answer A cannot be your answer. If J has SV as its only feature and P has RF, then answer B could be true. If J has HL as its only feature, then answer C could be true. If J and P have RF, then answer D could be true. Answer E is impossible because the only stops that can have HL are J, L, M, and P. Your deductions determined that K and N could not have HL. Therefore, the correct choice is answer E.

8. Answer: B

The question tells you that all the stops that have HL also have SV and asks you to determine which among the possible answers must be false. This means that all but one of the answers could possibly be true. The new information tells you that the second feature of P must be SV since it also has HL. Since P cannot have more than two features, it cannot have RF. Therefore, you can quickly determine that answer B must be false and it must be your choice. Answer A is your deduction from the new information and must be true. Answer C is the result of your original deductions and must be true. Answers D and E may or may not be true. J does not have HL, but that does not mean that it cannot also have SV. Likewise, you cannot determine whether it has RF or not. Since these statements cannot be determined as false or true, neither can be your answer. The correct choice is answer B.

Images

9. Answer: C

The question tells you that P and J have no features in common but that P has one feature in common with every other stop. Then, it asks you to determine which answer must be false. This means that all but one of the answers either must be true or could possibly be true. You apply the new information to your diagram. In order for P to have one feature in common with every stop other than J, it must have RF. If you assigned SV to P, then P would still not have a feature in common with stop N. Therefore, J must have SV, because it is the only remaining feature that P does not have. With the feature set determined for all the stops, it should be easy to find your answer. Answers A, B, D, and E are all reflected in the following diagram. Only answer C is impossible because J can only have one feature (SV) and K and N cannot have HL. The correct choice is answer C.

Images

10. Answer: B

The question tells you that no two stops have exactly the same features as one another and asks you to identify which of the answers cannot be true. All but one of the answers could be true. You apply the new information to your diagram. You need only worry about J and P, since they are the only stops whose features remain variable. For stop J, you can determine only that it does not have RF, because the only other stop with one feature is N. Therefore, J must have SV or HL. For stop P, the only other stop with two features is L, which has SV and HL. P already has HL, so it must have RF as its second feature. According to your diagram, answer A must be true. Because P must have RF and J cannot have RF, exactly four stops have restroom facilities. This also means that answer B must be false and must be your choice for the correct answer. Answers C and D are possible if J’s one feature is HL. Lastly, answer E is possible if J’s one feature is SV. The correct choice is answer B.

Images

11. Answer: E

The question tells you that exactly four stops have HL and exactly four have SV, then asks you to determine which answer could be false. That means all but one of the possible answers must be true. You apply the new information to your diagram. For four stops to have historic landmarks, J must have HL, because your deductions determined that K and N could not have HL. Because J must have HL, P must have SV in order to satisfy the second requirement that four stops have SV. With all the features set, it is just a matter of looking through the answers for the one that conflicts with your diagram. You can quickly see that answer E cannot be true. Since P must have SV, stops P and L have two features in common and cannot have exactly one in common. All the remaining answers match your diagram perfectly and must be true. The correct choice is answer E.

Images

12. Answer: C

This question changes the clues and gives you a different starting point for solving the game. It removes the requirement that M have more features than L and sets a new requirement, that M and L have the same two features. This change does not alter your other deductions, and it also does not change the possibility that M has all three features, but it does remove RF as a required feature for stop M. You remove the mark from RF for M, but you do not black that box out. The question asks you to determine which among the answers must be false. Answer A may or may not be true. Even under the new conditions, J may have any one of the three features, so it is possible that J and M have a feature in common. Since there is a possibility it can be true, this cannot be your choice. Answer B also could be true. The new conditions do not prohibit stop P from having SV and HL, which are the same features as M. By the same token, it is impossible that M and P have no features in common. Stop M must have SV and HL. Stop P must have HL, so even under the new conditions, M and P will share at least one feature. This must be your choice for the correct answer. Answer D must be true because K and M both must have SV. Answer E could be true if M also has RF. The correct choice is answer C.

Images

Question 13–18

As with all logic games, you follow the six-step process.

STEP 1:  Identify the game type.

This is an ordering game. You know this because you are being asked to determine the order of arrival of the seven buses.

STEP 2:  Begin your diagram.

There are seven buses that will arrive in order from first to seventh. You can create a grid with seven columns, and to make your job easier, you can abbreviate the names of the buses to their first initials. The arrivals go in order from left to right. For reference, write the list of buses in the upper right corner above your grid.

Images

STEP 3:  Symbolize the clues.

Symbolize the clues to make them easy to refer to as you work through the questions.

Clue 1: The buses arrive one at a time.

This sort of requirement is usually incorporated into the description of the game and is not listed among the clues. It does not require symbolization, but it tells you that no more than one bus can arrive at a time. If desired, you could put a number 1 below each column, indicating that each slot can only add up to one bus.

Clue 2: Either the Lynnville or the Koenig arrives fourth.

To indicate that either L or K but no other bus will arrive fourth, you put L/K above the fourth column.

Clue 3: The Fortman arrives at some time after the Koenig but at some time before the Lynnville.

This clue tells you that the F bus arrives between the K and the L buses. You can use the following notation to show this relationship:

K — F — L

Clue 4: Both the Hampton and the Janistown arrive at some time after the Evans.

You can use the following notation to symbolize this relationship:

E — H & J

Clue 5: The Hampton does not arrive next after the Janistown; nor does the Janistown arrive next after the Hampton.

Essentially, this clue is saying that the J bus and the H bus cannot arrive consecutively. You can represent this relationship using your box notation and the double arrow, as follows:

Images

STEP 4:  Double-check your symbolizations.

To double-check your symbolizations, you translate your symbolized clues back into normal English and see whether they match the original language of each clue. Once you have verified that they work, you can add your symbolizations to your diagram. Your diagram should look as follows:

Images

STEP 5:  Make deductions.

Before you tackle the questions, you attempt to make deductions based on the setup of the game and the clues. You go through each type of deduction.

1. Can’t-be-first-or-last deductions

Clue 3 tells us that L must come after F, so F cannot be last in the order. It also tells you that K cannot be last or even second to last because both F and L have to come after it. The opposite is true as well: bus F cannot be first and bus L cannot be earlier than third. You can represent these deductions in your diagram by placing the negation above the columns in which they cannot appear (see the diagram that follows).

Clue 4 tells you not only that E cannot be last but also that the latest it can be is fifth, because both H and J must come after it (this will change again later, but for now, you are focusing only on this particular deduction). This clue also tells you that neither H nor J can arrive first because E must arrive before both of them. You represent these deductions by placing the negation above the columns in which they cannot appear.

2. Repeated-element deductions

Buses H and J are repeated in clues 4 and 5. Clue 4, as we have seen, tells you that H and J must come after E. Clue 5 tells you that H and J cannot be consecutive in whatever order they arrive. There must be at least one bus between H and J. Therefore, the latest E can arrive is not fifth, as previously determined, but fourth, because there has to be another bus between H and J (for example, if J arrives fifth, then H must arrive seventh with another bus arriving sixth). You must put a ~E above the fifth column. Now you notice that E is restricted to arriving first, second, or third.

3. Down-to-two deductions

No deductions of this type are possible in this game.

4. Block-splitting deductions

No deductions of this type are possible in this game.

Your diagram with deductions added should look like this:

Images

STEP 6:  Answer the questions in the smartest order.

As you take the test, answer the questions in this order:

1. Answer the Complete and Accurate List question.

2. Answer questions that give you more information to work with.

3. Answer the remaining questions.

Question 13 is a Complete and Accurate List question and should be answered first. The questions that give more information are as follows:

Question 14 (“Koenig arrives at some time before the Evans . . .”)

Question 17 (“exactly one of the buses arrives after the Koenig but before the Lynnville . . .”)

Question 18 (“Delbert arrives at some time before the Evans . . .”)

Remaining are questions 15 and 16, and they can be answered in that order.

THE ANSWERS

13. Answer: D

The question asks you to identify the answer that gives an acceptable order for the buses to arrive. You can look at each answer in your diagram and evaluate whether it is consistent with your clues and deductions. You quickly eliminate answer A because neither L nor K arrives fourth. In answer B, bus H arrives first and that is inconsistent with clue 4 and your deductions. Answer C has buses H and J arriving consecutively, which is inconsistent with clue 5. Answer D is completely consistent with the clues of the game and is your choice for the correct answer. Answer E has bus L arriving before bus F, which is inconsistent with clue 3. The correct choice is answer D.

Images

14. Answer: A

The question tells you that bus K arrives before bus E and asks how many different orders are possible given this condition. You can create the various orders in your diagram. You know right off that L and not K must arrive fourth because the latest E can arrive is fourth, and if K must be earlier than E, then K must arrive earlier than fourth. Clue 3 tells you that not only K and E but also F must arrive before L. Therefore, K, F, and E must be the first three buses to arrive. K must be first because it must arrive before F (clue 3) and E (the question’s requirement). The only two of these buses that can change in the order are F and E, alternating between the second and third arrival. After the fourth arrival (bus L), clue 5 tells you that H and J must arrive fifth and seventh with D arriving between them. The only two of these that can change in order are H and J. As you see in the diagram that follows, there are only four orders with F and E alternating positions and H and J alternating positions. Since there are only four possible orders, the correct choice is answer A.

Images

15. Answer: B

The question asks that you choose the answer that must be true. This means that all but one of the answers could be false. Answer A could be true. Bus E could be first—nothing in the clues tells you otherwise—but it could just as easily be false, so this cannot be your answer. Answer B must be true. Clue 3 tells you that bus K must arrive before bus F and before bus L, so your choice must be answer B. You could stop there, but let’s look at the other choices for learning purposes. Answer C could be true or false since D is the one bus that can arrive pretty much at any time except fourth (K/L). Answer D could be true or false since there is no clue governing the relationship between the J and L buses. Also, the L bus has the freedom to arrive last, so the J or any other bus could easily arrive before it. Answer E could be true or false. This answer tries to confuse you by using the reverse order stipulated by the previous question. You must be sure to forget the information given by the previous question and return to the original game each time you assess a question. In this case, the E bus is restricted to arriving in the first three places, but the K bus has the freedom to arrive anytime between the first and fifth places. Bus E could arrive first and K second, or vice versa without contradicting your clues. Therefore, this answer cannot be your answer. The correct choice is answer B.

16. Answer: E

The question asks that you choose the answer that could be true, which means that all but one of the answers must be false. You can test each answer in your diagram. Answer A cannot possibly be true. If L arrives fourth, then clue 3 tells you that K and F must arrive before L in an order consistent with the clue (K-F-L). Buses E, H, and J are forced to arrive together in the fifth, sixth, and seventh slots. On the other hand, if K is the fourth arrival, clue 3 tells you that D, F, and L have to occupy the fifth, sixth, and seventh arrivals in a manner consistent with the clue and E, H, and J are forced to arrive together in the first, second, and third slots. Unfortunately, neither leaves the possibility for an order that is consistent with clues 4 and 5. In either situation, E, H, and J are forced to arrive consecutively, and clues 4 and 5 say this is not possible. Bus E must arrive before H and J, and H and J cannot arrive consecutively. Answer A cannot be the correct choice. Answers B and C have the same problem as answer A. The arrival of D after or before F in any configuration forces E, H, and J to arrive consecutively, so these cannot be your answer. Answer D could not possibly be true. Clue 4 says that E must arrive before H and J. The E bus cannot arrive after either of those buses or the order will be inconsistent with this clue. Finally, answer E could be true. You have already seen in question 14 that E can be the next bus to arrive after F. It was one of the four possible orders that were acceptable. The correct choice is answer E.

17. Answer: A

The question tells you that one of the buses arrives after K but before L and then asks you which of the possible answers could be true. That means all but one of the answers are false. If exactly one bus arrives between K and L, then clue 3 tells you it must be F. Buses K, F, and L must arrive consecutively. This creates a block and you can look at two different orders, one with K arriving fourth and the other with L arriving fourth. You can plot both of these possibilities in your diagram (marked 1 and 2). When K arrives fourth, answer A could be true because E can be scheduled first or second without causing an inconsistency with the clues. When L arrives fourth, E must arrive first. Therefore, this must be your choice for the correct answer. For learning purposes let’s assess the remaining answers. Answer B is not possible. Because K, F, and L form a block and clue 2 locks K or L into the fourth arrival, F must arrive either fifth or third. Answer C cannot be true either. When K arrives fourth, E can only arrive first or second. When L arrives fourth, E can only arrive first. Answer D cannot be true for the same reason as answer B: bus F can only arrive fifth or third. Finally, answer E cannot be true because H and J cannot be consecutive. When K is fourth, the K, F, and L block takes over the sixth arrival. When L is fourth, bus E is forced to be first, which means H and J must arrive fifth and seventh so that another bus may arrive between them. The correct choice is answer A.

Images

18. Answer: C

The question gives you a situation in which the D bus arrives before the E bus and asks you to determine when K arrives. For D to arrive before E, they must both arrive earlier than fourth, which leaves only one spot available for another bus to arrive before the fourth bus. If L arrives fourth, there are not enough slots available for both K and F, so K must arrive fourth. The correct choice is answer C.

Questions 19–25

As with all logic games you follow the six-step process.

STEP 1:  Identify the game type.

This is an ordering game. You know it is an ordering game because you are asked to arrange the trucks in sequential order at docks 1 through 7; and because you are asked to assign each truck to only one dock, it is a one-tiered ordering game.

STEP 2:  Begin your diagram.

The diagram for this game will be a simple grid with seven columns into which you will place your trucks. You can put the truck names in the upper right corner for easy reference.

Images

STEP 3:  Symbolize the clues.

You symbolize the clues as follows:

Clue 1: P is unloaded at a lower numbered dock than T.

Using your visualization of the docks, the clue says that P must be to the left of T. This can be symbolized using the line notation, as follows:

P — T

Clue 2: W is unloaded at the dock numbered one lower than the dock at which Q is unloaded.

This clue gives you the exact order of two trucks, but it does not say specifically where in the order they appear together. You use your box notation to show that they must be next to each other, with the W truck to the left of the Q truck.

Images

Clue 3: R is unloaded at dock 1 or else dock 7.

The clue says that R is located at 1 or 7. It also says that R cannot be unloaded at docks 2 through 6. You can represent this clue by writing ~R above columns 2 through 6.

Clue 4: Z is unloaded at dock 4.

The clue says truck Z is unloaded at dock 4, and you can represent this clue by simply putting Z in column 4 of your diagram.

STEP 4:  Double-check your symbolizations.

To double-check your symbolizations, you translate your symbolized clues back into normal English and see whether they match the original language of each clue. Once you have verified that your symbolizations work, you can add them to your diagram. Your diagram should look as follows:

Images

STEP 5:  Make deductions.

Finally, before you tackle the questions, you see if you can make any deductions based on the setup of the game and the clues. You go through each type of deduction.

1. Can’t-be-first-or-last deductions

Clue 1 tells you that P must come before T. Therefore, you can deduce that P cannot be last in the order. Also, T cannot be first since P must come before it. You can write ~T above column 1 and ~P above column 7.

Clue 2 tells you that W must come before Q. Therefore, you can deduce that W cannot be last in the order. Also, Q cannot be first since W must come before it. You can write ~Q above column 1 and ~W above column 7.

2. Repeated-element deductions

No deductions of this type are possible in this game.

3. Down-to-two deductions

Clue 3 already tells you that column 1 or 7 must unload truck R, but the second truck could be any of the other trucks, except those the other clues exclude. No further deductions of this kind are possible.

4. Block-splitting deductions

Clue 2 creates a block of W and Q. Also, truck Z must be at dock 4, so your diagram is split into two groups of three docks. Truck W cannot be unloaded at dock 3 because Q would have to be unloaded at dock 4, which is already occupied by Z. You can write ~W above column 3.

You add the deductions and your complete diagram should look as follows:

Images

STEP 6:  Answer the questions in the smartest order.

As you take the test, approach the questions in this order:

1. Answer the Complete and Accurate List question.

2. Answer questions that give you more information to work with.

3. Answer the remaining questions.

Question 19 is the only Complete and Accurate List question for this game. It should be answered first. Questions that give more information are as follows:

Question 21 (“U is unloaded on dock 5 . . .”)

Question 23 (“U is unloaded at dock 2 . . .”)

Question 24 (“Q is unloaded at a dock numbered one less than the dock at which P is unloaded . . .”)

Question 25 (“W is unloaded at dock 1 . . .”)

The remaining questions are 20 and 22, and they can be answered in that order.

THE ANSWERS

19. Answer: C

The question asks you to identify an acceptable loading assignment for the trucks among the possible answers. The following diagram depicts each answer choice. Answer A is unacceptable because P is loaded in a higher dock than truck T and this is inconsistent with clue 1. Answer B is unacceptable because Z is not unloaded at dock 4, which is inconsistent with clue 4. Answer C is consistent with all your clues and is your answer. Answer D is unacceptable because truck W and truck Q are not unloaded next to each other, which is inconsistent with clue 2. Answer E is unacceptable because R is not loaded at dock 1 or 7, which is inconsistent with clue 3. The correct choice is answer C.

Images

20. Answer: C

The question asks which answer gives the correct lowest numbered dock at which a specified truck can be unloaded. The trick is to test each answer and see whether the truck can be unloaded at a lower dock. Looking at your diagram you see that if R is unloaded at dock 7, then that makes room for P to be unloaded at dock 1, which is lower than dock 2. Therefore, answer A cannot be the correct choice. Again, if R is unloaded at dock 7, then W can be unloaded at dock 1 and Q can be unloaded at dock 2, which is lower than dock 3. Therefore, answer B cannot be correct. Considering answer C, your deductions allowed you to determine that T cannot be unloaded at dock 1. The lowest T can be unloaded is dock 2. Therefore, answer C must be the answer. Truck U is unrestrained by any of your clues. If R is unloaded at dock 7, U can be unloaded at dock 1 or dock 2, both of which are lower than dock 3. Therefore, answer D cannot be your choice. In considering answer B, you determined that W could be unloaded at dock 1, which is lower than dock 2. Therefore, answer E cannot be your choice. The correct choice is answer C.

21. Answer: A

The question tells you that truck U is unloaded at dock 5, then asks which of the pairs could be unloaded at consecutive docks, but not necessarily in the order given in the answer. This means that all but one of the pairs could not possibly be unloaded at consecutive docks. It also means that each answer creates a new block with which you must contend in each scenario. You add the new information to your diagram and assess each pair. With answer A, R can be unloaded at dock 7 and T could be unloaded at dock 6. See the following diagram for this configuration. This must be your answer, but let’s review the remaining options for learning purposes. The pair given by answer B, TW, is not possible. The pair cannot be at the higher docks because there would not be a dock to accommodate Q, which must be after W. They also cannot be located in the lower docks because they must accommodate P, which must come before T. If you unload P, T, and W at docks 1, 2, and 3, then again you do not have room for truck Q, which must be unloaded right after W. This answer cannot be correct. Answer C, WZ, is impossible. Because U is unloaded at 5, the only configuration is W at 3 and Z at 4 (clue 4), but this is impossible because clue 2 says that Q must unload at the next highest dock to W. This cannot be your answer. Answer D, QR, is not possible. Truck R must be at dock 1 or 7. This answer asks whether Q could be unloaded at dock 2 or 6. Neither is possible. If Q is at dock 6, then W must be at 5, but the question requires that U be unloaded at 5. If Q is at dock 2, then W must be at 1, which is impossible because R is unloaded at that dock. This cannot be your answer. Finally, answer E, QT, is not possible. Truck T must be loaded after P. If Q and T are unloaded at docks 2 and 3 (in whatever order), then P must be unloaded at dock 1, but this configuration leaves no room for Q to come after W. The same is true for docks 6 and 7. The correct choice is answer A.

Images

22. Answer: B

The question asks what is the most number of docks that can come between the docks at which P is unloaded and T is unloaded. You can test this by putting T at the highest dock (since it must come after P) and moving P around to see how far up you can move it. You unload T at dock 7, so R must be unloaded at dock 1. If you put W and Q at 5 and 6, you can put P and U at 2 and 3 respectively. This order is acceptable given the clues of your game. This is the farthest you can place P and T from each other. There are four docks between them. Therefore, the correct choice is answer B.

Images

23. Answer: D

The question tells you that U is unloaded at dock 2 and asks you to identify the statement among the possible answers that must be true. This means that all but one of the answers may or may not be true. You put the new information into your diagram and assess the answers based on what you see. Before you even look at the answers, you recognize that W and Q must be unloaded either at 5 and 6 or 6 and 7 to be consistent with clue 2, because those are the only consecutive loading docks available. Of course, this makes it impossible for both P and T to be to the right of Z. Actually, regardless of where T is unloaded, P must be to the left of Z, because there is only one remaining open dock to the right of Z. Answer A may or may not be true. Truck T can be unloaded at dock 3 as long as P is unloaded at dock 1 and R is unloaded at dock 7, so Z could be unloaded at a higher dock (4) than T (3). That means answer A cannot be correct. For the same reason, W could be unloaded at a higher dock (5 or 6) than T (3), so answer B cannot be your answer. If you unload truck R at dock 1, P at dock 3, and T at dock 7 (W and Q are at 5 and 6 respectively), then T could be unloaded at a higher dock than R. Therefore, answer C cannot be the correct choice. Answer D must be true. As you deduced, no matter where T is unloaded, P must be to the left of Z, so it must be unloaded at a lower dock. This is your answer. Answer E cannot be correct because no matter where W and Q are situated, you still have the option of unloading R at 1 or 7. If R is at 7, then it is being unloaded at a higher dock than Q. The correct choice is answer D.

Images

24. Answer: E

The question tells you that Q is unloaded at a dock numbered one less than the one that unloads P and then asks you to pick the answer that must be true. Because Q is part of the WQ block, the new fact creates an even larger block, WQP. In order for three trucks to be together, they must be at docks 1 through 3 or docks 5 through 7. But you also know that T must come after P. Therefore, the block cannot be unloaded at docks 5 through 7. They must be unloaded at docks 1 through 3. You can write these deductions into your diagram. Answer A cannot be true because W must be unloaded at dock 1. Answer B may or may not be true. U could be unloaded at 5 or 6. It is interchangeable with T. Answer C cannot be true because W is unloaded at dock 1. Answer D cannot be true because Q must be unloaded at dock 2. Finally, answer E must be true. Truck P must be unloaded at dock 3 for the order to be consistent with the new fact and your clues. The correct choice is answer E.

Images

25. Answer: B

The question tells you that W is unloaded at dock 1, then asks you which of the answers could be true. This means that all but one of the answers must be false. Since W is unloaded at dock 1, clue 2 tells you that Q must be unloaded at dock 2. Also, clue 3 tells you that because W is at dock 1, truck R must be at dock 7. You put the new information as well as your deductions into your diagram. There are still three docks unassigned. You now assess each answer. Answer A must be false. Truck U cannot be unloaded at a dock one lower than Q because that is reserved for W in order for the configuration to be consistent with clue 2. Answer B could be true because you can assign U to dock 3, P to dock 5, and T to dock 6. This order is consistent with your clues and the facts given by the question, so this must be the correct answer. For learning purposes, let’s review the remaining options. Answer C must be false because if T is assigned to dock 3, then P must be assigned to a higher dock than T and this is inconsistent with clue 1. Answer D cannot be true. Trucks P and U would have to be assigned to docks 5 and 6 respectively and T would have to be assigned to dock 3. This order is inconsistent with clue 1 because T would be assigned a lower dock than P. Answer E must be false. Since R is assigned to dock 7 and P to dock 6, T would be assigned a lower dock than P and this is also inconsistent with clue 1.The correct choice is answer B.

Images

SECTION II

1. Answer: D

STEP 1:  Read the question and identify your task.

This is a Describe question. It asks you to assess Kathy’s argumentative tactic in countering David’s argument.

STEP 2:  Read the argument with your task in mind.

Read both David’s and Kathy’s arguments. David argues that after an injury an athlete must relearn how to play the game in the manner that the athlete did before the injury. Kathy disagrees and tells a story about a professional quarterback who suffered an injury and afterward adjusted his game to a new style.

STEP 3:  Know what you’re looking for.

The correct answer will describe Kathy’s technique as one using a counterexample that undermines David’s argument.

STEP 4:  Read every word of every answer choice.

Answer A is incorrect because Kathy is not really establishing any solution. She is merely giving an example of a particular quarterback’s solution to his problem. Also, the term solution seems wrong. Neither David nor Kathy is offering a solution. Rather, they are offering or suggesting a prescription. The difference is subtle but enough of a reason to exclude this as a possible answer. Answer B cannot be correct because at the outset it is obvious Kathy disagrees with David. She does not support his claim. Regarding answer C, there is no equivocation in David’s argument. He makes a fairly straightforward statement that the only way an athlete can be successful after an injury is to relearn the athlete’s original method of play. Kathy is equally unequivocal and uses stark language to disagree. Answer D is exactly what you formulated as the correct answer. Kathy does in fact use an example to counter David’s claim, so this is most likely your answer. Answer E cannot be your answer. David and Kathy are not trying to explain the same phenomenon. On the contrary, they are offering a different path to success. The correct choice is answer D.

2. Answer: A

STEP 1:  Read the question and identify your task.

This is a Conclusion question. The question asks that you identify the answer that most accurately states the main point of the argument.

STEP 2:  Read the argument with your task in mind.

You read the argument, which states that digital camera manufacturers advertise a particular feature for their cameras but evidently this feature is irrelevant because it is incompatible with other technologies and thus can be considered impractical.

STEP 3:  Know what you’re looking for.

The central assumption of the argument maintains that the memory capacity of digital cameras typically is far in excess of electronic screen capabilities, so differences in memory are not really significant.

STEP 4:  Read every word of every answer choice.

Answer A seems to be giving exactly the meaning of the argument, that the resolution of the cameras’ sensors has no bearing on their relative quality as tools. This would seem to be your answer, but you must review the remaining options to be sure. Answer B says the argument is making a recommendation that manufacturers add more features to their cameras, but the argument never makes such a recommendation. The argument focuses entirely on resolution as an advertised feature and its inherent problems. This cannot be your answer. Answer C says the argument is making an assessment of the manufacturers’ decision to focus on resolution, but the argument is only describing the flaw in that decision. Such an assessment may be implied or the next step, but what you have before you makes no judgment regarding their decision. This cannot be the correct choice. Answer D is incorrect because it focuses on the last part of the argument and ignores the manufacturers and their choice of what to emphasize in advertising. Finally, answer E cannot be the answer because it suggests that resolution is the only practical difference between the cameras. Also, the argument does not make a claim concerning the difference in quality between cameras. It only claims that the emphasis on resolution is misplaced as a quality measure. The correct choice is answer A.

3. Answer: A

STEP 1:  Read the question and identify your task.

This is an Assumption question. It asks which answer gives the assumption upon which the argument depends.

STEP 2:  Read the argument with your task in mind.

The assumption is that the memory capacity of the camera is not a critical concern because modern electronic screens and photographic paper transfer methods cannot convey a majority of the data captured.

STEP 3:  Know what you’re looking for.

The answer should contain wording that makes a connection between the very high data memory capacity of digital cameras and the much lower resolution abilities of electronic screens and other methods related to image display.

STEP 4:  Read every word of every answer choice.

Essentially, answer A states that differences in resolution do not affect the deficiencies in screens and paper. The argument does in fact argue that resolution is irrelevant because the deficiencies in screens and paper limit the level of detail. Thus, the argument depends on resolution having no effect on those deficiencies, which is what this answer states. This is most likely your answer, but you must review the remaining options to be sure. Answer B cannot be correct because the argument bases its claim on technology, in particular the resolution and its interaction with screens and photographic paper, not on the skill of the photographer. Answer C may be true but your argument does not even mention the software involved in displaying digital photographs on screens. It is uncertain that software is even part of the equation, so you cannot choose this as your answer. Answer D actually contradicts part of the argument by stating that resolution determines the degree of detail reproduced. The argument makes the opposite argument. Answer E makes a strange connection between the definition of the term resolution and its effect on judging the quality of photographic instruments. The argument does not necessarily depend on the definition of the term resolution. Rather, it depends on how the measure of resolution is used by manufacturers and their claims’ relationship to practical uses. This cannot be your answer.The correct choice is answer A.

4. Answer: C

STEP 1:  Read the question and identify your task.

This is a Conclusion question. The question asks you to identify the statement supported by the argument.

STEP 2:  Read the argument with your task in mind.

The argument is about a misconception that a large breakfast decreases calorie intake for the rest of the day when it actually makes no difference whatsoever.

STEP 3:  Know what you’re looking for.

The correct answer might support a statement that says eating a large breakfast either makes no difference or counteracts efforts to reduce calories.

STEP 4:  Read every word of every answer choice.

Answer A is not supported by the information. The argument implies that the size of breakfast has no effect on calorie intake outside of breakfast. This means the calorie intake outside of break-fast is fixed or unchanged. On the other hand, this also implies that the size of the breakfast may have beneficial or detrimental effects, and a small breakfast versus no breakfast may in fact have different dieting benefits. This cannot be your answer. Answer B may give an explanation for why the non-breakfast food calorie intake remains unchanged, but nothing in the argumentsupports that notion. It is equally possible that they eat lower-calorie food for the rest of the day but more of it, thus overall calorie intake remains the same. There is nothing in the argument that allows you to choose one theory over the other. Answer C says that a small or no breakfast can reduce calorie intake while a large breakfast increases it. The argument supports this statement because it says that the size of the breakfast has no effect on calorie intake for the remainder of the day, but this means that the size of the breakfast may very well increase or decrease overall calorie intake. This is most likely your answer, but you should review the remaining answers to confirm the wisdom of your choice. Answer D says the size of the breakfast makes no difference, but the argument specifies that it will make no difference to calorie intake for the rest of the day. It does not indicate that the size of the breakfast will have no overall effect. In fact, it implies the opposite (answer C). Answer E has the same fault as answer B. It may give an explanation for why the non–breakfast food calorie intake remains unchanged, but nothing in the argument supports that notion. It is equally possible that they eat less of higher-calorie foods. The correct choice is answer C.

5. Answer: D

STEP 1:  Read the question and identify your task.

This is a Flaw question. The question asks that you determine why the argument is questionable.

STEP 2:  Read the argument with your task in mind.

The argument describes a survey covering three cities in the Midwest and from the results argues that the national mortgage default rate cannot be attributed to homeowners’ lack of desire to pay off their mortgages.

STEP 3:  Know what you’re looking for.

You can expect the correct answer to point out some flaw in the survey or the conclusion that the survey of the Midwest justifies a conclusion regarding the entire nation.

STEP 4:  Read every word of every answer choice.

Answer A cannot be your choice because the argument does not include two conclusions, conflicting or not. The argument makes only one conclusion, which is that the high mortgage default rate is not due to a lack of desire on the part of the homeowners to pay off their mortgages. Answer B cannot be your choice because 92 percent is an overwhelming majority by any expert or nonexpert’s standards. Answer C cannot be your choice because the high default rate does not contradict what could be considered a desire on behalf of those homeowners to pay off their mortgages. They are two very different statistics, one based on actual behavior, the other based on what people want for the future. Answer D fits into your expectations for the correct answer. It points out that the argument is using a survey of the Midwest as a basis for a statement regarding the entire nation. This seems to be your answer, but you must review the final option. Answer E cannot be your answer because the argument does not overlook this possibility. The argument says if the default rate is high, and it is only concerned with this possibility. The implication is that the opposite is possible but not of concern in this particular argument. The correct choice is answer D.

6. Answer: C

STEP 1:  Read the question and identify your task.

This is a Strengthen question. The first question asks you to identify the principle or axiom that supports Professor Raleigh’s side of the argument.

STEP 2:  Read the argument with your task in mind.

Professor Allyn argues that political speeches are not true literary works and should be excluded from the literature department’s curriculum. Professor Raleigh believes that regardless of whether they are considered true literary works, political speeches should be included because the department can give their students the skills to help them be better citizens in the larger society.

STEP 3:  Know what you’re looking for.

You expect the correct answer to be a principle supporting the department’s responsibility to make their students better citizens through teaching them to better analyze political speeches.

STEP 4:  Read every word of every answer choice.

Answer A cannot be correct because it is too general and does not even address the specifics of their argument. Also, both professors would probably agree with this principle, and you are being asked which answer supports only Raleigh’s argument. Professor Raleigh might agree with answer B, but it is a principle that assumes speeches are already taught in the department. Essentially, it puts the cart before the horse. Answer C says pretty much what you formulated as your expected correct answer. It says that the literature department should enable students to handle all texts that have an effect, dangerous or otherwise, on society. This is most likely your answer, but you should review the remaining answers. Answer D is supportive of Raleigh’s position and contradicts Allyn’s position, but it is a bit too general and does not address the subject of political speeches. Answer E supports a claim that Raleigh does not make. Raleigh says that whether political speeches are a literary form or not they should still be taught. Therefore, this principle does not fit Raleigh’s argument. The correct choice is answer C.

7. Answer: A

STEP 1:  Read the question and identify your task.

This is a Assumption question. It asks that you identify the assumption upon which Raleigh bases his argument.

STEP 2:  Read the argument with your task in mind.

Raleigh’s argument seems to be based on the idea that the literature department should be teaching students something beyond just true literary works, so you expect your answer to be a similar idea.

STEP 3:  Know what you’re looking for.

The correct answer will state that the literature department’s responsibility includes more than teaching true literary works.

STEP 4:  Read every word of every answer choice.

Answer A says that a literature department’s academics should not be limited to analyzing true literary works. Raleigh’s argument depends on the idea that the literature department should be doing more than just teaching true literary works since he is advocating that they teach about political speeches whether they are considered literature or not. This option is most likely your answer, but you should continue to review the remaining answers. Answer B is incorrect because Raleigh talks about all political speeches and does not make a distinction regarding the circumstances surrounding the political speech. Answer C focuses on Raleigh’s statement concerning the goal of the department’s courses, but this is a rather limited assumption and Raleigh’s argument does not exactly depend on it. If this assumption proved false and courses taught other skills, it would not diminish his argument concerning political speeches. In fact, it might strengthen it. Answer D cannot be correct because Raleigh’s argument considers political speeches important regardless of whether they are considered true literary works. Therefore, since the literary status of political speeches is irrelevant to his argument, then whether true literary works are detrimental or not to society is irrelevant as well. Finally, answer E cannot be the answer because it actually states the opposite of what Raleigh is arguing. The correct choice is answer A.

8. Answer: B

STEP 1:  Read the question and identify your task.

This is a Principle question. The question asks you to identify which answer most accurately illustrates the principle expressed in the argument.

STEP 2:  Read the argument with your task in mind.

The argument states that a child follows a teacher’s instructions better if given a few options rather than many.

STEP 3:  Know what you’re looking for.

The correct answer will most likely restate this principle in a different manner while remaining consistent with the intent of the argument.

STEP 4:  Read every word of every answer choice.

Answer A cannot be your answer because it ignores the many-versus-few options aspect of the argument and states that success is a matter of defining the benefits of each option. This is certainly not in the same spirit of the argument. Answer B states that there is an inverse relationship between children’s change of behavior and the number of alternatives given them, so children will alter their behavior (abide by instructions) more if they have fewer choices and alter their behavior less (refuse to abide by instructions) if they have more choices. This is exactly what your argument is saying and is most likely your answer, but you must review the remaining options. Answers C, D, and E focus on the delivery of instructions and not the content of them. Your argument is more concerned with the content, mainly how many options, so none of these answers can be correct. The correct choice is answer B.

9. Answer: E

STEP 1:  Read the question and identify your task.

This is an Assumption question. The question asks you to identify the assumption upon which the argument depends.

STEP 2:  Read the argument with your task in mind.

The argument tells you about a fungus threatening crops of tomatoes and a synthetic fungicide that can control it. The fungicide is harmful to humans, so it cannot be used in populated areas. Then the argument makes the conclusion that the nation’s tomato crops are not threatened by the fungus because commercial tomato fields are not located near populated areas and the fungicide can be used on them.

STEP 3:  Know what you’re looking for.

You can guess that the correct answer will have something to do with equating the nation’s tomato crops with commercial tomato fields, since its final statement is based on that assumption.

STEP 4:  Read every word of every answer choice.

Regarding answer A, the argument is concerned only with the threat of late blight on tomato crops, but it does not depend on it being the only threat. The existence of other threats would not diminish the argument in any way. Answer B is concerned with a possible future, but the argument is concerned only with the current threat. This cannot be the correct choice. Answer C discusses the speed with which the fungus spreads through particular crops, but the argument is not based on the speed of the attack. Instead, it is based on the fungicide used to stop it and its side effects. This cannot be correct. Answer D is irrelevant because the argument is about the commercial farms and the nation’s tomato crops. It is not about these smaller sources of tomatoes. Finally, answer E states that commercial fields produce most or all of the nation’s tomatoes. This is what you formulated as your possible answer. The argument’s main assumption is that the commercial fields are equal to the nation’s tomato crops. Without that assumption, the conclusion that the nation’s tomato crops are not seriously threatened is not possible. The correct choice is answer E.

10. Answer: D

STEP 1:  Read the question and identify your task.

This is a Flaw question. The question asks that you identify the false presumption at the heart of the argument.

STEP 2:  Read the argument with your task in mind.

The lawyer predicts that the next objection will be overruled by the judge based on a statistic.

STEP 3:  Know what you’re looking for.

You can guess that the probable answer will identify the false presumption as that of using a statistic to solidly predict the next result.

STEP 4:  Read every word of every answer choice.

Answer A cannot be the presumption because no such requirement guiding the judge’s ruling is mentioned in the argument and such a presumption would eliminate the word probably from the lawyer’s argument. It would make the overruling a certainty. Answer B cannot be the presumption because the lawyer’s prediction is based on past behavior, plain and simple, not a more complicated expectation based on the influence past decisions had on the verdicts in those cases. Answer C cannot be the presumption because the lawyer says the next objection will “probably” be overruled, which implies likelihood, not a guarantee. A presumption of a guarantee would lead the lawyer to state that the objection will be overruled. Also, the lawyer only states that the next object will likely be overruled, not the next five as the answer presumes. Answer D could very well be your presumption. It is very similar to answer C, but the lawyer does presume the likelihood of the next objection when he or she says it will “probably” happen. Also, this presumption only concerns itself with the next objection, which is exactly the lawyer’s prediction. This is most likely your answer, but you have one last option to consider. Answer E cannot be the presumption because the lawyer’s prediction is based on the judge’s decisions and not on the quality of those decisions. The correct choice is answer D.

11. Answer: B

STEP 1:  Read the question and identify your task.

This is an Assumption question. The question asks that you identify an assumption that justifies the conclusion of the argument.

STEP 2:  Read the argument with your task in mind.

The argument states that as a percentage of tax revenues, the cost of maintaining the national parks is half today what it was 50 years ago. It tells you that tax revenues rose after the last 50 years and then concludes that tax revenues rose at a greater rate than the cost of maintaining the parks. The argument moves from rate of growth to actual growth and back to rate of growth. If actual tax revenues grew, the only way that the rate of growth of tax revenues could be higher than the rate of growth of maintenance costs would be if the actual costs of park maintenance increased very little or not at all.

STEP 3:  Know what you’re looking for.

You would expect the correct answer to be along this line of thinking.

STEP 4:  Read every word of every answer choice.

Answer A states that the parks are in better condition and less costly to maintain today than they were 50 years ago. Nothing in the argument suggests that the costs of maintenance have decreased. It says only that the percentage of revenues has decreased. Also, the conclusion suggests an increasing growth rate of both maintenance cost and tax revenues, so your assumption should suggest something to do with growth, not reduction. Answer B is similar to your expected answer. It suggests that the parks require the same maintenance today as 50 years ago. This might very well be your answer, but you must review the remaining options. Answer C is completely irrelevant since the maintenance of federal buildings is not even part of the argument. Answer D tries to confuse you by focusing on another type of rate, breaking down the cost to “per square mile” of the national parks, but the argument is concerned only with the total national expenditures on national parks. It is possible that the cost per square mile remained identical while the total area of national parks decreased or increased (no telling by how much). Answer E is similar to answer C, focusing on an irrelevant fact, the cost of maintaining “other federal properties.” The correct choice is answer B.

12. Answer: C

STEP 1:  Read the question and identify your task.

This is a Describe question. The question asks you to choose the answer that describes the argument that the critic makes in response to the mayor’s argument.

STEP 2:  Read the argument with your task in mind.

The city mayor argues that an increase in percentage of students graduating from high school is proof that the mayor’s education policies are a success. The critic argues that another metric concerning preparedness for college and jobs proves the opposite.

STEP 3:  Know what you’re looking for.

You expect the correct answer to state that the critic uses new data to show that the mayor’s data leads to the wrong conclusion.

STEP 4:  Read every word of every answer choice.

Answer A is incorrect because the critic sticks to the numbers and says nothing about the mayor personally or the mayor’s motives. Answer B cannot be your answer because the critic uses a statistic to support his or her conclusion, so the critic obviously believes that statistics have credibility and that measuring progress toward education goals is not difficult. Answer C says the critic attempts to show that the mayor’s use of facts is misleading. This is indeed what you expected the answer to say. The critic uses a statistic to show that the mayor’s facts lead to the wrong conclusion. This is most likely your answer, but you must review the remaining options. Answer D is tempting and the critic’s attack might be read that way, but the critic does not exactly discredit the mayor’s methods so much as undermine the mayor’s argument in support of his methods. Also, this description ignores the use of statistics within the argument. Answer E cannot be your choice because the critic does not dispute the mayor’s statistics. The critic points out only that the mayor’s facts do not lead to the right conclusion. The correct choice is answer C.

13. Answer: D

STEP 1:  Read the question and identify your task.

This is a Flaw question. The question asks you to identify a fact that the mayor could use to counter the critic’s claim.

STEP 2:  Read the argument with your task in mind.

You expect the correct answer will undermine the basis for the critic’s argument, which is that the test indicates the students’ lack of preparedness.

STEP 3:  Know what you’re looking for.

The correct answer will make the case that the test is inadequate in some manner.

STEP 4:  Read every word of every answer choice.

Answer A points out that the students who tested poorly were located in a particular region, but this does not invalidate the test or its use as support for the critic’s argument. Answer B is irrelevant since the critic is not using the students’ eventual career in college or in the work world as a basis for the argument. Answer C is a statement the critic might use to undermine the mayor’s argument, not the other way around, so this cannot be your answer. Answer D undermines the test by saying that it is new and the results this year cannot be compared to anything, which, if true, undermines the use of this test in the argument since the mayor makes his argument based on statistics over the several years he has been in office. This is probably your answer, but you have one more option to consider. Answer E is irrelevant since both the mayor and the critic are concerned only with their own city and no other city. The correct choice is answer D.

14. Answer: D

STEP 1:  Read the question and identify your task.

This is an Assumption question. The question asks that you identify the assumption upon which the argument relies.

STEP 2:  Read the argument with your task in mind.

The argument says that bleached flour is priced much higher than unbleached flour even though the process to bleach the flour is not very costly. It concludes that the price difference must have to do with something other than the cost of “providing” bleached flour to the consumer.

STEP 3:  Know what you’re looking for.

You can expect the assumption to be that the cost of bleaching is the only cost that should be considered when attributing causes for the price difference.

STEP 4:  Read every word of every answer choice.

Answer A is somewhat outside the scope of the argument because it focuses on consumer demand. The argument suggests that something other than cost may be responsible but does not indicate what that may be. The argument bases its conclusion on cost. Also, this assumption indicates that consumer demand should force bleached and unbleached to be priced the same. Answer B is also outside the scope of the argument. It might be a conclusion you draw from the argument, but it is not an assumption upon which the argument is based. Answer C is a clarification or furthering of the conclusion, and it is not an assumption upon which the argument is based. Answer D states that the cost of the bleaching process is the only factor relevant to the cost of providing bleached flour to the consumer. This is probably your answer, but you have one more option to consider. Answer E seems to actually contradict the argument and thus cannot be an assumption upon which it is based. The correct choice is answer D.

15. Answer: A

STEP 1:  Read the question and identify your task.

This is a Conclusion question. It asks you to identify the main conclusion that can be drawn from the argument.

STEP 2:  Read the argument with your task in mind.

The argument states that a newspaper article’s claim that lawsuits are on the decline is based on a misleading statistic, court verdicts, and that one should consider alternative methods of resolving lawsuits.

STEP 3:  Know what you’re looking for.

You can expect the correct answer to be that by considering these other methods it becomes clear the article’s claim is not true, lawsuits are not on the decline.

STEP 4:  Read every word of every answer choice.

Answer A states this conclusion exactly. This is very likely your answer, but you must review the remaining options to be sure. Answers B and C may be true, but nothing in the argument gives you a cause-and-effect relationship between the length and expense of court hearings and the growth of arbitration and mediation. The argument tells you only that the hearings are a result of the failure of these other means. The success or failure of lawsuits through court hearings is not even discussed in the argument. The argument is concerned only with the length and expense of the process, so answer D cannot be your choice. Answer E has to be true for the argument to work, but it is not the conclusion. Rather it is a precondition. The correct choice is answer A.

16. Answer: D

STEP 1:  Read the question and identify your task.

This is a Describe question. It asks that you identify the strategy used by the argument.

STEP 2:  Read the argument with your task in mind.

The argument looks at the basis for the claim in the article, the decreasing number of court verdicts, and says that this statistic can be understood differently by looking at it in a wider context and taking into consideration alternative methods of resolving disputes, thus leading to a very different conclusion.

STEP 3:  Know what you’re looking for.

You expect the correct answer to give a succinct description of this approach.

STEP 4:  Read every word of every answer choice.

Answer A is incorrect because the argument makes no claim regarding the motivation of the journalists. The argument sticks to attacking the journalist’s argument itself. Answer B obviously cannot be your answer because the argument does not explore the history of lawsuits and alternative dispute resolution. Like the newspaper article, the argument concentrates on the current situation. Answer C also cannot be correct because the argument says nothing about the relationship between lawyers and judges. Answer D says exactly what the argument does and fits your expectations of the correct answer perfectly. The argument does in fact look at the evidence used by the article and reinterprets it to reach the opposite conclusion. This is most likely your answer, but you have one last option to consider. Answer E cannot be your answer because the argument does not question the statistic or its accuracy. Instead, the argument accepts the statistic as true but questions how the statistic is interpreted. The correct choice is answer D.

17. Answer: A

STEP 1:  Read the question and identify your task.

This is a Flaw question. The question asks that you identify the one answer that fails to point out a flaw in the president’s reasoning. This means that all the options but one are valid flaws in the president’s reasoning.

STEP 2:  Read the argument with your task in mind.

The president is questioning the vice president about a manager’s decisions. Based on the manager’s hiring the best engineer and the best production team and the best staff, the president questions the vice president’s assessment that the manager did not want the new product to succeed.

STEP 3:  Know what you’re looking for.

The correct answer will either have nothing to do with the president’s logic or it will actually support his criticism of the vice president.

STEP 4:  Read every word of every answer choice.

Answer A does not seem to describe the president’s logic at all, nor does it describe a flaw in the president’s logic. The relationship between the manager’s desires and the success of the product is not at issue. What’s at issue is the judgment of the vice president regarding the manager. This is most likely your answer, but you must review the remaining options to be sure. Answer B essentially says the president fails to acknowledge that the vice president misinterpreted the facts and made a poor assessment of the manager. Therefore, the vice president did not willfully misrepresent the manager’s performance. With regard to the president’s accusation against the vice president, this is definitely a flaw in the logic and cannot be your choice. Answer C is also a flaw. The president does assume the manager hired the engineer, team, and staff upon his own will, so this could be considered a flaw because if the manager had no choice, then the manager could very well have wanted the new product to fail despite the nature of the hires. Answers D and E are also valid flaws. They both point out factors outside the hires. Despite the quality of those hires, if the manager failed to allot enough time and resources or if the manager knew the hires would not work well together, these facts would indicate that the manager wanted the new product to fail. The correct choice is answer A.

18. Answer: B

STEP 1:  Read the question and identify your task.

This is a Parallel question. The question asks you to match the flawed reasoning in the argument with the answer that exhibits similarly flawed logic.

STEP 2:  Read the argument with your task in mind.

Essentially, the argument says a person likes X, but not Y. Therefore, it is not true that any person who likes Y likes X. You also notice that the statements go from the specific (my brother) to the general (whoever). You are looking for the same pattern among your options.

STEP 3:  Know what you’re looking for.

The correct answer will follow the pattern described in step 2.

STEP 4:  Read every word of every answer choice.

Answer A says that a person likes X but not Y. Therefore, it is not true that any person who likes X likes Y. Notice that the second half of the pattern has been reversed (XY instead of YX). This answer does not match your pattern. Answer B says a person is X (7+) but not Y (a giant). Therefore it is not true that any person who is Y (a giant) is X (7+). Even though the wording is somewhat different, the flawed logic follows the same pattern and this is most likely your answer. You should review the remaining options to be sure. Answer C says all compositions that are X (minimalist) are Y. Therefore if a composition is X, then it is Y. Again, the pattern is not the same, plus the order is reversed, moving from the general to the specific while the argument goes from the specific to the general. Answer D says people who do X are Y. Therefore, the neighbors who are Y are X. The order of terms is correct but this option, like answer C, also goes from the general to the specific, which is the wrong direction of logic. Finally, answer E says a photo that is not X is Y. Therefore, it is not true that some photos that are X are Y. The order is wrong and the logical terms are wrong. The first term is in the negative. Also, the general statement is not categorical (all, every, etc.) but “some.” The correct choice is answer B.

19. Answer: D

STEP 1:  Read the question and identify your task.

This is a Weaken question. The question asks that you determine which fact most undermines the basis for the expert’s conclusion.

STEP 2:  Read the argument with your task in mind.

The expert says that they tested 5 square inches of an explosive substance 10 times, and every time it failed to explode an area beyond 20.7 square feet. The expert’s evidence is that the tests exploded areas between 9.2 and 12.5 square feet. The expert concludes that 5 square inches of the substance explodes an area much less than 20.7 square feet.

STEP 3:  Know what you’re looking for.

You expect the correct answer to be some fact that counters the evidence presented by the tests and offers proof that the explosive may explode an area close to or beyond 20.7 square feet.

STEP 4:  Read every word of every answer choice.

Answer A is about testing another type of explosive in identical circumstances, but this does not undermine the expert’s evidence since another explosive tells you nothing about the explosive used in the bank robbery. Answer B has nothing to do with the evidence and attempts to undermine the expert’s qualifications. This is not what the question is asking. Answer C also tries to undermine the expert’s qualifications by describing a common malpractice, but it is still not relevant to the evidence at hand and it is too general a statement to be of use. Answer D says that another test was done that still did not cover 20.7 square feet, but it came close at 20.1 feet. This seems close enough to lay some doubt about the expert’s use of only the 10 previous tests and the conclusion that the substance destroys an area “much less” than 20.7 square feet. This may be your answer, but you have one more option to consider. Answer E is a hypothetical and may be true, but it is not as strong as answer D at undermining the expert’s evidence and conclusion. The correct choice is answer D.

20. Answer: A

STEP 1:  Read the question and identify your task.

This is an Assumption question. The question asks that you identify an assumption upon which the argument depends.

STEP 2:  Read the argument with your task in mind.

The argument claims that the Internet increases the awareness of injustices for those attempting to solve injustices. Then it says that Internet access comes faster than an enlightened education system, and that people are vulnerable to questionable reformers promising solutions. Thus, countries with enlightened technology policies may cause their own downfall.

STEP 3:  Know what you’re looking for.

The correct answer will most likely connect poor education to a lack of ability to recognize fake reformers.

STEP 4:  Read every word of every answer choice.

Answer A says that a lack of enlightened education affects the ability of people to recognize between legitimate and illegitimate reformers. This matches your expectations for the correct answer exactly and is most likely your answer, but you must review the remaining options to be sure. Answer B offers a direct causal relationship between the education level and the toppling of the nation, but the argument says that only some countries, not all, may experience such a connection, and it definitely does not make its claims with such certainty. Answer C supports the opposite of what the argument is saying. Answer C says that a charlatan can topple a government only if there is an enlightened education system in place, but the argument says that it is the lack of an enlightened educational system that enables charlatans to achieve such a result. The argument says that Internet allows more people to be aware of injustices, but it does not say that the Internet is the only source, so answer D cannot be your answer. Finally, answer E makes a connection between enlightened education and the sustaining of reasonable regimes. The argument connects a lack of enlightened education with the toppling of reasonable regimes. It makes no claim regarding the opposite condition, so this cannot be correct. The correct choice is answer A.

21. Answer: A

STEP 1:  Read the question and identify your task.

This is a Deduction question. The question asks you to identify the statement that is most strongly supported by the argument.

STEP 2:  Read the argument with your task in mind.

The argument states that sleep aids increase appetite and that some of the resulting weight gain can be counterbalanced by dieting.

STEP 3:  Know what you’re looking for.

You can expect the correct answer to be related to the connection between sleep aids and gaining weight.

STEP 4:  Read every word of every answer choice.

Answer A says that some patients who take sleep aids gain weight as a result. This matches your expected answer. Nothing in the statement is unsupportable by the argument, and this most likely is your answer, but you must review the remaining options to see if there isn’t a better one. Answer B states a recommendation in categorical terms, but the argument does not say that all people who take sleep aids gain weight, nor does it say that dieting helps all people gain weight. This cannot be your answer. Answer C is unsupportable by the argument because the argument links the weight gain to taking the drugs, not to dieting or lack of dieting. Answers D and E are incorrect because the argument does not go into the decision of whether or not to prescribe. Nor does it go into a patient’s decision to request the drug. These elements are beyond the scope of the argument, which is concerned only with those who have already taken the drugs and how they deal with the resulting weight gain. The correct choice is answer A.

22. Answer: B

STEP 1:  Read the question and identify your task.

This is a Conclusion question. The question asks that you identify the answer choice that restates or clarifies the conclusion of the argument.

STEP 2:  Read the argument with your task in mind.

The argument says that education and training enable people to use their intelligence efficiently and that successful people do not vary much in intelligence. Therefore, to achieve success, people must be well educated.

STEP 3:  Know what you’re looking for.

You can guess that the correct answer will link education to success more succinctly than the argument does.

STEP 4:  Read every word of every answer choice.

Answer A is a partial restatement of the first premise in the argument, but it is not a restatement of the conclusion, so it cannot be your answer. Answer B says that no person can be successful without a superior education. This is pretty much a restatement of the conclusion and is close to what you expected. The argument does conclude that in order for a person to be successful, he or she must have a good education. This is probably your answer, but you should go through the remaining options. Answer C attempts to reverse the relationship, saying that only successful people have a superior education, but the argument says a superior education is a prerequisite for success and does not say that a superior education is exclusive to successful people, nor does it say that all people with a superior education become successful. Answer D is simply a restatement of the second premise in the argument and is not related to the conclusion. Answer E tries to connect intelligence to success, but the argument says there is not much difference in intelligence among successful people. It concludes that education impacts success, and it is this conclusion that you are being asked to find restated among the answers. The correct choice is answer B.

23. Answer: E

STEP 1:  Read the question and identify your task.

This is a Principle question. The question asks you to identify the principle or axiom that most justifies the claim made in the argument.

STEP 2:  Read the argument with your task in mind.

The argument says that you should avoid attacking an opponent’s character before a football game and that such attacks have nothing to do with competitiveness; rather, they question the team’s moral right to compete.

STEP 3:  Know what you’re looking for.

You expect the principle to state that attacks that are not related to competitiveness are irrelevant or something to that effect.

STEP 4:  Read every word of every answer choice.

Answer A attempts to say that attacks on character fail to achieve a moral result, but the argument is saying that such attacks should not be used in the first place, so this cannot be your answer. Answer B says that attacks on character are warranted in certain cases, but the argument makes it clear that such attacks are never warranted. Answer C may be true, but the perception of the attacks by the spectators is not the concern of the argument, so this cannot be correct. Answer D offers a theory as to why teams attack the character of the opposing team, but this is an explanation, not a justification for the argument. Finally, answer E states your expected answer perfectly. If such behaviors should be avoided, then definitely attacks on character, which the argument says have nothing to do with competing on the field, should be avoided. The correct choice is answer E.

24. Answer: C

STEP 1:  Read the question and identify your task.

This is a Parallel question. The question asks that you identify the argument that uses the same logical reasoning as the main argument.

STEP 2:  Read the argument with your task in mind.

The argument gives you a bureaucratic situation that says the manager cannot post an opening (A) without the superior’s approval (B), but the superior cannot approve it unless Human Resources approves it first (C). You can reorder these as “if C, then if B, then A.”

STEP 3:  Know what you’re looking for.

The answer must follow the same pattern.

STEP 4:  Read every word of every answer choice.

For answer A, each attribute exists concurrently with the others, and one is not required before the other, so this cannot be correct. Answer B has conditionals that are not certain. They are likelihoods. Language such as “most patients” and “it is likely” disqualify this answer from consideration since the argument’s requirements are certainties. Answer C says a student (A) cannot achieve a brown belt without achieving a purple belt (B), but the student cannot achieve the purple belt without first achieving a blue belt (C). In other words, if C, then if B, then A. This is the same pattern as in the argument and is likely your choice, but you must review the remaining options. Answer D has two alternative paths to reach the ultimate goal while the argument has only one path, so this cannot be correct. Answer E says a microscope “can” be used to examine the flagellum, but it does not say the microscope is the only way to examine the flagellum, so there might be other tools that can be used to do so. Therefore, this cannot be your answer. The correct choice is answer C.

25. Answer: E

STEP 1:  Read the question and identify your task.

This is an Assumption question. The question asks you to identify the assumption upon which Morrel’s defense depends.

STEP 2:  Read the argument with your task in mind.

Morrel argues that he is not guilty of plagiarism because Stipich made his movie in Russian and it was never subtitled.

STEP 3:  Know what you’re looking for.

You can expect the correct answer to be something to the effect that Morrel had no other way than viewing the original film, subtitled or not, to learn the plot.

STEP 4:  Read every word of every answer choice.

Answer A cannot be your choice because Morrel does not defend himself by saying he knew of the plot from some other source. Answer B might indicate that Morrel might be able to figure out the language in the movie, so this would actually make him look more guilty and would certainly not be an assumption underlying Morrel’s claim of innocence. Answer C cannot be correct. Morrel does not base his defense on being unable to understand Stipich if and when they ever met. Answer D cannot be correct because the popularity of Stipich’s movie is irrelevant to whether Morrel plagiarized the movie. Answer E fits your expected answer well. Morrel is assuming that there was no other way for him to learn plot elements of Stipich’s movie than by viewing the film, so he is also assuming that nobody told him the plot of Stipich’s movie before he made his movie. This is your answer. The correct choice is answer E.

SECTION III

Passage 1

1. Answer: C

STEP 1:  Read the question and identify your task.

This is a Main Idea question. The question asks that you identify the answer that gives you the main idea of the passage.

STEP 2:  Go back to the passage to find the answer.

Refer to your passage summary.

STEP 3:  Read every word of every answer choice.

Answer A offers the opposite of what the passages are saying. The passage discusses nations attempting to maintain both Western and traditional legal systems, not attempting to supplant traditional with a Western-style system. Answer B is tempting because it discusses the efforts of the countries to create constitutions that satisfy the competing interests, but it does not mention the difficulties the countries have faced in attempting to integrate Western and traditional legal systems, and this is a central aspect of the passages. Answer C is better. It talks about how nations are working toward Western-style legal systems, but their efforts to balance competing interests have encountered conflicts with traditional legal structures. You must review the remaining options to see if there is a better one. Answer D is too focused on legal cases while the passage is more general. Also, the passage discusses how the rule of law moved legal cases back to traditional, more informal institutions after the court system proved inadequate to the task. Finally, answer E is too limited to the last paragraph and the accusation by human rights groups. It leaves out the rest of the passage. The correct choice is answer C.

2. Answer: D

STEP 1:  Read the question and identify your task.

This is a Main Idea question. This question asks for the main idea of just the second paragraph of passage A.

STEP 2:  Go back to the passage to find the answer.

Look back to that particular paragraph and read the first and last sentences to recall that paragraph’s purpose.

STEP 3:  Read every word of every answer choice.

Answer A cannot be your answer because the paragraph does not suggest that these legally pluralistic nations have been highly successful; if anything, it suggests that they have been problematic. Answer B cannot be correct because the paragraph does not discuss the history of legal pluralism or different forms it has taken over time. Answer C cannot be correct. While the paragraph does discuss the difficulties of achieving legal pluralism, it does not make a prognosis that it is unworkable. Answer D describes paragraph 2 very well. The first paragraph discusses the issue of legal pluralism in general, while paragraph 2 presents Africa as a place where the abstract problem has actually arisen. This is probably your answer, but you have one last option. Answer E is a bit extreme. It does not say that the results have been “disastrous” throughout history. In the reading-comprehension section you should always be suspect of answers that use such extreme words or descriptions of any passage. The correct choice is answer D.

3. Answer: A

STEP 1:  Read the question and identify your task.

This is an Inference question. The question asks you to identify the attitude newly independent governments have toward traditional leaders as implied by the passage.

STEP 2:  Go back to the passage to find the answer.

There is little specific information in the passage regarding their attitude, but the passage does give the impression of governments earnestly trying to integrate the two, so there is a willingness on their part to create a system of legal pluralism that works.

STEP 3:  Read every word of every answer choice.

Answer A seems like a good description of a government trying to integrate traditional values into a wider world value system even though it is difficult to do so and the attempt does not always work very well. To be sure that this is your answer, however, you must review the other options. Answer B could be our choice. The word “accommodation” looks right, but the “disdain” is a bit harsh. Disdain requires evidence that the traditional leaders are coercing or pressuring the governments to create legal pluralism and that is not evident in this passage. Answer C is also too extreme and has the same problem as answer B. There is no evidence that the governments are being forced to create legal pluralism. They are just trying to work out a compromise. If the governments felt anger and resentment they would probably be much less willing to attempt legal pluralism in the first place. Answer D would require that the traditional leaders gave the governments reasons to be suspicious or uneasy, and none of those elements are explored here. Lastly, answer E is wrong because if there was anything to be uncertain or anxious about it would be Western legal systems, not their own traditional leaders. They might be uncertain or anxious about integrating the two systems, but not the traditional leaders. The correct choice is answer A.

4. Answer: C

STEP 1:  Read the question and identify your task.

This is a Line ID question. The question asks that you identify the reason the passage uses the example of Zimbabwe’s attempts to reform its water management system.

STEP 2:  Go back to the passage to find the answer.

Find the discussion of this subject, which is in the second paragraph of passage B. This example seems to be used to further exhibit the difficulties governments face at creating legal pluralism when they must accommodate traditional interests. You expect the correct answer to say something along those lines.

STEP 3:  Read every word of every answer choice.

Answer A might be correct if the example described a success, specifically the creation of a new water management system, but the example does not do this. Instead, it shows how such an effort did not succeed, at least not at first. Answer B cannot be correct because the example does not explore the different approaches to water management. You are given no specifics. It says only that the conflict existed because informal systems and traditional institutions controlled water systems. Answer C is most likely your answer because the example does come after another example of the difficulties with legal pluralism, and it is far from similar to the first example. Thus, it offers insight into the range of difficulties. You must review the remaining options to be sure this answer is correct. Answer D mentions “foreign interests,” and nothing in the example indicates that foreign interests were involved in the attempt to achieve reform of the water management system. Answer E is tempting, but the example says only that a conflict ensued. It says nothing about the end result, so you do not know whether there was a failure or not. The correct choice is answer C.

5. Answer: D

STEP 1:  Read the question and identify your task.

This is a Main Idea question. In the case of paired passages, look for the points of comparison between the main ideas of passages A and B.

STEP 2:  Go back to the passage to find the answer.

Refer to your passage summaries. Note that passage A discusses legal pluralism in general, whereas passage B discusses problems specific to Zimbabwe.

STEP 3:  Read every word of every answer choice.

Passage B does not reject any contention made by passage A, it only elaborates upon passage A, so you can reject answer choice A. Similarly, passage B never questions the validity of any statements made in passage A, so you can reject answer choice B as well. Answer choice C is tempting but too extreme. Passage B does not suggest that legal pluralism is the cause of a “global human rights conflict.” We know that pluralism has caused some problems, but not on the scale suggested by answer choice C. Answer choice D, however, correctly identifies our relationship. Passage A discusses legal pluralism in general, and passage B discusses the specific example of Zimbabwe. Passage A never discusses human rights, so you can eliminate answer choice E. The correct choice is answer D.

Passage 2

6. Answer: C

STEP 1:  Read the question and identify your task.

This is a Main Idea question. The question asks that you identify the main idea of the passage.

STEP 2:  Go back to the passage to find the answer.

Refer to your passage summary.

STEP 3:  Read every word of every answer choice.

Answer A uses the first few sentences of the first paragraph as a basis for its idea, but the passage does not say those critics are misinterpreting the novel. It just says that the reader could read the novel from a very different perspective than the one offered by those critics. Answer B seems to contradict what the passage is saying by centering its idea on the southern novel. The passage discusses critics who view Absalom, Absalom! as exploring the nature of the novel in general, not just the southern novel, so this cannot be your answer. Answer C is very promising. The passage does discuss the novel’s interpretation as an exploration of southern social structures and some critics’ view that it is also an exploration of the novel form and whether it can hold any truths. This is most likely your answer, but you should review the remaining options to be sure. Answer D may be tempting since the passage says that the novel might be calling into question the novel as a form of storytelling, but the passage does not suggest that the novel’s nature makes it impossible to interpret, nor does it suggest that critics have failed to do so. In fact, the passage is giving validity to just such an interpretation. Finally, answer E cannot be correct because the passage mentions the Hollywood circumstance only incidentally, and it is not a central element of the passage’s discussion. The correct choice is answer C.

7. Answer: B

STEP 1:  Read the question and identify your task.

This is an Information Retrieval question. The question asks that you identify the statement with which the author would agree concerning the contemporary critics.

STEP 2:  Go back to the passage to find the answer.

Look back at the discussion of the contemporary critics.

STEP 3:  Read every word of every answer choice.

The author would most likely not agree with answer A because the author states that the new critics recognize the central story but they argue that the reader can ignore it. It would be inaccurate to say that they fail to take it into account. Answer B looks promising. The author points out the influences that led to the novel’s structure but does not attribute these to the critics, implying they might not have taken these into account, but the author does believe the structure and end of the novel give validity to their theories. This may be your answer, but you should review the remaining options to be sure. Answer C says the opposite of what the author might say. The author seems to acknowledge that the critics have a valid perspective, so the author would surely not be so critical. Also, this is one of those answers that sounds too extreme to be the right answer. Answer D tries to confuse you by attributing the author’s observations to the critics. The author makes the connections to Hollywood and detective novels but does not attribute these influences to the critics’ analysis. Finally, regarding answer E, the author does not state that the new critics have overthrown previously accepted theories. Instead, the author merely suggests that the new critics are offering a different theory, one that fits into the modernist way of thinking. Never does the author suggest that previous theories have been supplanted. The correct choice is answer B.

8. Answer: B

STEP 1:  Read the question and identify your task.

This is a Main Idea question. The question asks you to identify the purpose of the second paragraph.

STEP 2:  Go back to the passage to find the answer.

Review the second paragraph. It gives some biographical information on Faulkner at the time that he wrote the novel as well as some outside circumstances that might have influenced the structure of the novel.

STEP 3:  Read every word of every answer choice.

Answer A cannot be correct because the paragraph does not attempt to cast doubt on the new critical theory or any theory for that matter. If anything, the paragraph seems to be supporting the new theory by discussing how the influences affected the structure of the novel. Answer B seems right. As we have already discussed, the second paragraph gives biographical information that could be seen as supportive of the new structuralist critical theory. This is probably your answer, but you must review the remaining options to be sure. Answer C cannot be correct because the paragraph is about Faulkner, not the critics or their process. Answer D is tempting and would finish a strong second to answer B, but the paragraph is not really focused on providing a historical background. Lastly, answer E is also a tempting answer because the paragraphcould be viewed as discussing the foundations of the theory, but the language is a bit too general when compared to answer B or even answer D. The word “foundations” could be interpreted too many ways, while “biographical information” is more specific and more accurate. The correct choice is answer B.

9. Answer: D

STEP 1:  Read the question and identify your task.

This is an Inference question. The question asks what the passage implies with regard to Faulkner’s main character in the novel, Quentin Compson.

STEP 2:  Go back to the passage to find the answer.

Find within the passage the section that discusses this character and reread it.

STEP 3:  Read every word of every answer choice.

Answer A cannot be correct because the passage never implies that Quentin is not from the South. It does the opposite, in fact, by noting Quentin’s “own southern heritage.” Answer B cannot be correct because the second paragraph (as we’ve already explored) implies that it was the structure of the novel that was influenced by Faulkner’s time in Hollywood, not the characters of the novel. Answer C may or may not be true. The passage does imply that Quentin’s account of what happened to the Sutpens indicated such a hatred—otherwise his roommate would not have asked the question—but the passage never indicates his true feelings, so this cannot be the correct choice. Answer D is indeed implied by the passage when the author writes that Quentin is Faulkner’s “frequent proxy,” which implies that the character appears in other novels and that he represents Faulkner’s perspective. This is most likely your answer, but you have one more option to consider. Answer E has the same problem as answer B: the detective novels influenced the structure, not the characters. The correct choice is answer D.

10. Answer: E

STEP 1:  Read the question and identify your task.

This is an Information Retrieval question. The question asks you to identify a statement that the modern critics would agree with.

STEP 2:  Go back to the passage to find the answer.

If you read the entire passage, you should be able to answer this question, but returning to the first and last paragraphs may help as well, since those paragraphs address specifically how the modern critics view the novel.

STEP 3:  Read every word of every answer choice.

Answer A is a statement that the older, more traditional critics would make, because they emphasize the story central to the novel, specifically the Sutpen family and how they represent the decay of the southern paternal system. Answer B suggests that the reader is a participant along with Quentin and his roommate in solving the mystery, but the passage does not discuss the reader’s role in the mysteries in the novel. Readers are discussed only with regard to being challenged about how they perceive the novel or novels in general. Answer C may or may not be true, but this answer goes beyond anything in the passage. The critics propose that the writer challenges readers concerning the nature of the novel, but they do not go as far as to suggest that there is a collaboration going on. The only collaboration mentioned in the passage is between Quentin and his roommate. Answer D cannot be your answer because it is the lack of such details that give the critics fuel for their theory. Finally, the critics would very likely agree with answer E because it states exactly (as discussed in the passage) what the critics think Faulkner was trying to achieve, a structure that allows readers to question whether what they are reading is important or true and to wonder what is the nature of the novel as a medium. The correct choice is answer E.

11. Answer: D

STEP 1:  Read the question and identify your task.

This is an Information Retrieval question. The question asks that you identify the one element that according to the passage did not influence Faulkner’s novel.

STEP 2:  Go back to the passage to find the answer.

You refer to the passage and scan for all the influences mentioned. You should be able to eliminate from the answer choices all those that are specifically included in the passage.

STEP 3:  Read every word of every answer choice.

Answer A is mentioned in the first part of paragraph two. Answer B is mentioned in the latter part of paragraph two. Paragraph one discusses the southern aspects of the novel, giving support to answer C. Answer D may be true, but the passage does not discuss gothic story structure, so this may be your answer. You need to consider your last option. Answer E is not as clear as the others, but the passage says that Quentin is Faulkner’s proxy, so it is implied that Faulkner also has such struggles. The correct choice is answer D.

12. Answer: B

STEP 1:  Read the question and identify your task.

This is a Main Idea question. The question asks you to identify the main purpose of the author in writing the passage.

STEP 2:  Go back to the passage to find the answer.

If necessary, you can review the passage by reading the first and last sentences of each paragraph to get a sense of the author’s purpose.

STEP 3:  Read every word of every answer choice.

Answer A concentrates only on paragraph two and thus cannot be your choice because it ignores the rest of the passage that discusses the critical theory. Answer B is closer to the author’s intent. The author is definitely exploring a critical theory and why Faulkner structured his novel to allow that theory to come to the fore. This is most likely your answer, but you need to go through the remaining options to be sure. Answer C comes from the first paragraph, but the author does not say that the approaches are necessarily in conflict, nor that there is an attempt to resolve them. Answer D focuses on the last paragraph but excludes all that came before, so this cannot be your answer. Finally, answer E has a similar problem to answer A. It concentrates on paragraph two. The correct choice is answer B.

Passage 3

13. Answer: E

STEP 1:  Read the question and identify your task.

This is a Main Idea question. The first question for this passage asks you to identify the main idea of the passage.

STEP 2:  Go back to the passage to find the answer.

Review the first and last paragraphs to get a sense of the main idea.

STEP 3:  Read every word of every answer choice.

Answer A cannot be correct because the passage suggests that budgetary conditions may make it difficult, but the passage does not suggest that it is impossible. When an answer uses such categorical terms as never, you can almost always eliminate that answer. Answer B cannot be correct because the passage never discusses a disagreement between the two different theories. It merely presents the different theories and posits that future research could reconcile them. Nor does the passage suggest that any of the theories are hindering progress. Only the lack of money is hindering progress, according to the passage. Answer C cannot be correct because the universe’s future is not unresolved. The passage states for certainty that it is expanding and not contracting. Answer D cannot be correct because Einstein’s theory did not necessarily fail to deliver an explanation. The passage makes it clear that one of his failed theorems offers a possible explanation, so this cannot be your choice for the answer. Finally, answer E seems to give a good explanation of the main idea. The passage is suggesting that until tests are done, scientists are left with a theoretical concept (dark energy) and several hypotheses about how it works within the universe. The correct choice is answer E.

14. Answer: A

STEP 1:  Read the question and identify your task.

This is an Information Retrieval question. The question asks you to identify the reason that the author mentions the known quantity of the energy necessary to sustain the current rate of expansion in the universe.

STEP 2:  Go back to the passage to find the answer.

Reread those particular lines in the passage (at the end of paragraph one).

STEP 3:  Read every word of every answer choice.

Answer A seems like a good answer. It is true that the passage is suggesting that scientists understand the overall universe and what is necessary to sustain its expansion, but they hardly know what is enabling it to do so, so this is probably the correct choice. Still, you must review the remaining options to be certain. Answer B is true but limited in scope. It focuses on what is not known, not on why the passage mentions what is known about the universe. Therefore, this cannot be your answer. Answer C cannot be correct because at that point in the passage the expansion of the universe is considered a given. The author is not still trying to prove that point. Answer D is incorrect because the author has not yet gotten to the discussion of the theoretical models that explain dark matter. Finally, answer E is also a bit premature since at this point you do not know the estimated magnitude of dark energy’s makeup of the universe. Knowing the total energy needed to expand the universe gives no indication of what makes up that energy. The correct choice is answer A.

15. Answer: C

STEP 1:  Read the question and identify your task.

This is an Inference question. The question asks you to identify what statement regarding quintessence can be inferred from the passage.

STEP 2:  Go back to the passage to find the answer.

Look back at the passage and review the section that discusses quintessence, or at least scan for key words in that section.

STEP 3:  Read every word of every answer choice.

Answer A cannot be correct because quintessence is not presented as contradictory to Einstein’s theory. The passage presents quintessence as one hypothesis that might explain the behavior of dark energy within Einstein’s cosmological constant. Answer B cannot be true because the conclusion of the entire passage is that quintessence and other ideas are just theoretical, and until observational evidence is obtained, we will never know the nature of the universe. Answer C says that dark energy is not inherent in space and is not consistent throughout the universe. The passage implies both of these ideas. It says that quintessence conceptualizes dark matter as a force that evolves with the growing universe. Also, quintessence conceptualizes dark matter as an “inhomogeneous energy fluid or field” that is not consistently present. This is most likely your answer, but you should review your remaining options. Answer D cannot be correct because the scientists have used Einstein’s cosmological constant to explain the excess energy. The excess energy is not a problem with Einstein’s equations as the answer suggests. Finally, answer E is too certain about what dark energy or quintessence has replaced or explained. Since it is still theoretical, you cannot yet accept such certainty even if the results seem likely. The correct choice is answer C.

16. Answer: B

STEP 1:  Read the question and identify your task.

This is a Main Idea question. The question is asking you to identify the primary function of the passage, or in other words, what the author is trying to do by writing the passage.

STEP 2:  Go back to the passage to find the answer.

Read the first and last sentences of each paragraph to get a sense of the progression of thought.

STEP 3:  Read every word of every answer choice.

Answer A does not seem right because the author does not really say the theories are in conflict with each other. The author only discusses various views that are held by different scientists. It is possible that each side considers the other theory just as likely to be correct as its own. Answer B is a good description of the passage. The author is presenting theories of dark energy, and while the author does not exactly discuss any challenges to the theories, it is evident that the theories are still under question by the scientific community. The author also says that these theories will remain theoretical until observational evidence is brought to bear. This most likely should be your choice, but you must review your remaining options. Answer C cannot be correct because the passage only discusses theories that are still considered viable until contrary evidence is found. They have not yet been discarded. Regarding answer D, even though the passage discusses problems with the cosmological constant theory, it does not favor quintessence over that theory. It merely presents it as an alternative theory. Answer E focuses on the last paragraph or conclusion of the passage, but it ignores the remainder of the passage, so this cannot be your answer. The correct choice is answer B.

17. Answer: E

STEP 1:  Read the question and identify your task.

This is an Information Retrieval question. The question asks that you identify the answer that is not a theoretically possible characteristic of dark energy.

STEP 2:  Go back to the passage to find the answer.

You refer to the passage and review the characteristics of dark energy.

STEP 3:  Read every word of every answer choice.

Answer A is what Einstein’s cosmological constant argues is the nature of dark energy when it says that dark energy constitutes the “energy density of the vacuum of space.” Answer B is mentioned in paragraph three, which says that quintessence theorizes that dark energy “evolves with the growing universe and exerts increasingly expansionary forces.” Answer C is mentioned in the first and last paragraphs, when the passage discusses dark energy as making up more of the universe and turning dark matter and matter into “minority players.” Also, the paragraph says that “the notion that gravity is the dominant force in the universe is no longer sustainable.” Answer D is a characteristic of the quintessence theory. Lastly, consider answer E. The passage says in the first few sentences that this characteristic of dark energy is no longer held to be true, so the correct choice is answer E.

18. Answer: B

STEP 1:  Read the question and identify your task.

This is an Inference question. The question asks you to identify a hypothetical sentence the author might include at the end of the last paragraph.

STEP 2:  Go back to the passage to find the answer.

Read the final paragraph to remind yourself of where it was leading you. The last two sentences discuss the potential for scientific exploration to discover the nature of dark energy, but suggest that financial limitations may kill such opportunities.

STEP 3:  Read every word of every answer choice.

Answer A does not fit the tone of the article, which is focused on the theoretical even if it is realistic about the prospects of finding evidence. Nothing in the article hints that scientists might just give up on proving their theories. Answer B fits the tone of the author. It is hopeful but realistic about the prospects of finding evidence of dark energy. This is most likely your answer, but you must review the remaining options to be certain of your choice. Answer C makes a rather unjustified statement about the public and its priorities. Nothing in the passage and especially the final paragraph supports such a harsh judgment. Answer D is completely off the mark. The passage states that the cosmological constant and quintessence are the scientific community’s best bets for explaining dark energy, at least until another theory or hard evidence is found. Nothing in the passage suggests that the author believes scientists should just abandon those theories. Finally, answer E might be a good sentence to come at the end of paragraph three if paragraph four did not exist, but this is not the case. Therefore, the correct choice is answer B.

Passage 4

19. Answer: D

STEP 1:  Read the question and identify your task.

This is a Main Idea question. The question asks that you choose a title for the passage from the possible answers.

STEP 2:  Go back to the passage to find the answer.

Scan the passage again or read the first and last sentences of each paragraph to get a sense of the passage’s meaning.

STEP 3:  Read every word of every answer choice.

Answer A’s title is not appropriate because Brandeis and Warren proposed privacy in the defense of personality, not property. Answer B’s title is somewhat valid. Warren and Brandeis did move privacy as a legal issue further into public consideration, but this title does not seem to be describing the overall idea of the passage, which is much more concerned with how they defined the legal issue of privacy and what it covered with regard to personality versus property. Answer C’s title ignores the issue of privacy altogether and focuses on creativity and copyright law. Although creativity was affected by Brandeis and Warren’s efforts, it seems wrong not to address privacy in the title since this is the central focus of the passage. Answer D addresses the issue of privacy and Brandeis and Warren’s approach to the issue with a humorous quip about personality. Since the title includes the two major elements discussed in the passage, this could very well be the title of the passage. You have one last option to consider. Answer E might be a close second place, but it focuses too much on the ultimate extent of Brandeis and Warren’s argument rather than on the overall argument that is discussed throughout the passage. The correct choice is answer D.

20. Answer: D

STEP 1:  Read the question and identify your task.

This is an Inference question. The question asks that you identify a statement regarding Brandeis and Warren’s approach to the privacy issue that can be inferred from the passage.

STEP 2:  Go back to the passage to find the answer.

You refer to the passage and read through the first paragraph, which discusses the origin and basis of their work.

STEP 3:  Read every word of every answer choice.

Answer A may be true and is a viable choice, but the passage actually says that the legal profession had defined privacy according to property and was satisfied with that definition. The implication is that Brandeis and Warren decided to redefine the law, not correct deficiencies. While this could be your answer, you cannot be certain and need to review the remaining options. Answer B cannot be your answer because it says that the concern was property, not personality, which was their basis for changing the law. Answer C is one of those extreme answers that you can reject altogether. Nothing in the passage implies such hostility or desire for revenge. Regarding answer D, the passage says that Warren enlisted Brandeis because he was “peeved” about gossip about higher society, and since Warren was very likely a member of high society, it is likely he was seeking a way to neutralize efforts to slander or blacken his and his friends’ reputation. This could very well be your answer, but you have one last option to consider. Answer E cannot be your choice because no such relation is even hinted at in the passage. The correct choice is answer D.

21. Answer: A

STEP 1:  Read the question and identify your task.

This is an Inference question. The question asks that you identify a statement that scholars other than Warren and Brandeis would agree with.

STEP 2:  Go back to the passage to find the answer.

Since Warren and Brandeis believed privacy extended to personality and not just property, you can expect the correct answer to favor defining privacy protection as a matter of protecting only property.

STEP 3:  Read every word of every answer choice.

Answer A discusses the stealing of a short story (written by person A) and the subsequent publishing of that story. The short story could be considered private property, and publishing it could be seen by the other scholars as an infringement of person A’s privacy because the property has been stolen word for word. This is most likely your answer, but you must review the remaining options to be certain of your choice. Answer B says the opposite of answer A. The photograph could be considered property, and stealing it and publishing it would be the same as the story in answer A, but this answer says the scholars would not consider it an infringement of privacy, which they actually would. This cannot be your answer. Answer C describes a person who steals the essence of someone else’s story and publishes his or her own version of the story. Since the exact story was not published, it is not the property of the original writer, so this would not be a breach of privacy according to the other scholars. Of course, Warren and Brandeis would very much consider this a breach under their more open definition of privacy that considers an unpublished work as an extension of a person’s personality, but that is not what the question asks. Answer D is a situation that covers gossip, which is the problem Warren was trying to solve in the first place. Obviously, other scholars thought this was acceptable behavior. Regarding answer E, the first person willingly gave the photograph to the second person, and it is unlikely that any scholar, Warren, Brandeis, or otherwise, would consider this an infringement of privacy. The correct choice is answer A.

22. Answer: D

STEP 1:  Read the question and identify your task.

This is an Information Retrieval question. The question asks you to identify how the passage portrays Brandeis and Warren’s approach to personality.

STEP 2:  Go back to the passage to find the answer.

Look back at paragraphs three and four, which discuss their approach to personality.

STEP 3:  Read every word of every answer choice.

Answer A cannot be correct because the passage says that Brandeis and Warren were influenced by trends in society, not by earlier ideas, which were focused on property. The passage actually implies that their ideas were a stark departure from earlier ideas. Answer B suggests that their approach was a tweak, but the passage really implies that the definition of privacy as protecting personality was a bigger change than just a tweak. Answer C seems completely off the mark. Their approach to personality was central to their argument and was definitely not a side step. Answer D seems like a good description of their approach as portrayed in the passage. The passage treats their approach to personality as a significant event that changed the legal definition of privacy going forward. This is most likely your answer, but you have one more option to consider. Answer E suggests that existing scholars poorly understood their approach, but nothing in the passage suggests such a problem. The passage does not really discuss the reception by existing scholars, so this cannot be your answer. The correct choice is answer D.

23. Answer: C

STEP 1:  Read the question and identify your task.

This is a Main Idea question. The question is asking you to identify how the second paragraph functions within the passage.

STEP 2:  Go back to the passage to find the answer.

You refer to the passage and review the second paragraph to assess its purpose. Reading just the first sentence, you get the sense that the second paragraph gives some historical context to Warren and Brandeis’s approach to privacy.

STEP 3:  Read every word of every answer choice.

Answer A might be correct if you focused on just one or two sentences in the paragraph, but there is much more to the paragraph than just the essay, and while the paragraph does mention some elements within the essay, there is not much clarification of those elements in that paragraph. Answer B cannot be correct because the paragraph does not even address the structure of the essay other than to say that much of it is devoted to the right to personality. Answer C is an appropriate description of the paragraph’s function. The paragraph discusses the rise of yellow journalism and the beginning of the Museum Period. These could be considered cultural phenomena that influenced Brandeis and Warren. This is most likely your answer, but you have two more options to consider. Answer D is actually a description of the first paragraph, not the second. Answer E cannot be your answer because part of the first paragraph functions this way and there are no personal details about the writers in the second paragraph. The correct choice is answer C.

24. Answer: E

STEP 1:  Read the question and identify your task.

This is a Main Idea question. The question asks you to identify the main purpose of the passage.

STEP 2:  Go back to the passage to find the answer.

Read the first sentences of each paragraph and the last sentence of the last paragraph to gain an overall understanding of the passage’s purpose.

STEP 3:  Read every word of every answer choice.

Answer A belongs to the “too extreme” type of answer. The passage does not seek to discredit or support any legal argument. It is merely discussing Warren and Brandeis’s efforts to redefine a legal concept. Answer B is tempting because the passage does suggest that their essay was historically influential on the definition of privacy up to today, but the passage is not mainly focused on describing that influence and therefore this cannot be your answer. Answer C has the same problem as answer B; the passage does indicate in the last paragraph that their essay had historic importance, but you cannot say this is the main purpose of the entire passage since there is so much content devoted to other aspects of the essay, its origin, its cultural context, and how its privacy argument contrasted with the previous definition of privacy. Answer D takes into account only the third paragraph and not the rest of the passage. Finally, answer E gives you what you need. The passage does indeed tell the circumstances and the nature of Brandeis and Warren’s historic legal argument. The correct choice is answer E.

SECTION IV

1. Answer: D

STEP 1:  Read the question and identify your task.

This is a Describe question. It asks you to identify the technique used in the argument.

STEP 2:  Read the argument with your task in mind.

Arbus argues that to make a 3-D version of a movie does not degrade the original version because a home video version of a movie is not considered a degradation of the original theatrical version; it is considered a different production altogether and is treated as such.

STEP 3:  Know what you’re looking for.

The correct answer will point out that Arbus makes his argument by likening one case to an analogous case that supports a different conclusion.

STEP 4:  Read every word of every answer choice.

Answer A cannot be correct because Arbus does not cite some general artistic or critical principle that guides the situation. Answer B cannot be correct because Arbus does not even discuss the popularity of 3-D movies. He addresses only the criticism itself. Answer C has the same problem as answer A. There is no general principle at work within the argument. Answer D says exactly what you expected the correct answer to say. The argument works by using an analogous situation to show the flaw in the criticism. This is most likely your answer, but you have one more option to consider. Answer E cannot be correct because the argument makes no such distinction between facts and value judgments. Also, the argument discards the value judgment in the criticism of 3-D movies but accepts it in the analogous situation with home video, so this cannot be your answer. The correct choice is answer D.

2. Answer: A

STEP 1:  Read the question and identify your task.

This is a Paradox question. The question asks you to identify the answer that helps resolve the discrepancy in the argument.

STEP 2:  Read the argument with your task in mind.

The argument discusses an international economic body deciding to help bail out a country. The prime minister of a member nation commits his nation to help out with the bailout. The parliament of his country condemns its prime minister for that commitment but supports the bailout.

STEP 3:  Know what you’re looking for.

You can expect the correct answer to resolve the discrepancy by saying that the parliament had a problem with its prime minister making the commitment without some sort of approval from the people of the country or the parliament itself.

STEP 4:  Read every word of every answer choice.

Answer A gives you a close approximation of what you need. It says the parliament is the only official body of that nation that can make such a commitment. This is probably your answer, but you should review the remaining options to be certain this is the right choice. Answer B may be true, but as a logical argument it is weakly constructed and does not resolve the discrepancy anywhere near as strongly as answer A, which is constitutionally enforceable. Answer C has the same problem as answer B in that it is a weaker resolution than the constitutionally enforced answer A. Answer D would be a good choice if the discrepancy existed between the G-20 and the member nation, but the discrepancy is between the parliament’s support for the bailout and lack of support for the prime minister, so this cannot be your answer. Finally, answer E is a logistical issue and is irrelevant to the discrepancy between support for the effort and lack of support for the prime minister. The correct choice is answer A.

3. Answer: E

STEP 1:  Read the question and identify your task.

This is a Parallel question. The question asks you to choose the answer that most resembles the flawed reasoning in the argument.

STEP 2:  Read the argument with your task in mind.

Essentially, the argument says that a game requires a particular skill. Because a woman is skilled in a profession (which you are meant to assume requires the same skill), she would make an excellent participant in that game.

STEP 3:  Know what you’re looking for.

The correct answer will follow the same pattern.

STEP 4:  Read every word of every answer choice.

Answer A says a person who performs one activity will be good at another, so a woman who is good at the one activity could learn to be good at the other. This is not the same pattern at all, especially since the logic orients between activities and not skills. Also, saying that someone can learn an activity is not saying the person will be good at it. This cannot be your answer. Answer B says that a particular activity is performed by a person with a particular skill. Because a man has been engaged in that activity for several years, he must have that skill. This is not your pattern either because it is saying the person will have the skill as evidenced by participating in the activity rather than saying he will be good at the activity as evidenced by having the skill or participating in a similar activity that requires that skill. Answer C says that people who have a certain skill invariably enjoy an activity. Because a man has a certain profession, he must enjoy that activity. This does not follow your pattern either. The activity does not require the skill and there is no claim that because the man is in a particular profession he would be very good at the activity. Answer D says that people with a physical attribute are good at an activity and that everyone in a woman’s family has that physical attribute so she must be good at that activity. The use of a physical attribute instead of a skill is problematic, and just because the woman’s family has the attribute does not necessarily mean that she has the attribute, so this cannot be your answer. Finally, answer E says a profession requires a skill. Then it says that a man participates in another profession (which you are to assume requires the same skill). Finally, it says that he will be good in the first profession. This is very close to the logic pattern in the argument and must be your answer. The correct choice is answer E.

4. Answer: B

STEP 1:  Read the question and identify your task.

This is a Flaw question. The question asks you to find the one answer that is not a flaw in the psychologist’s reasoning. This means that four out of the five possible answers are flaws in the psychologist’s reasoning.

STEP 2:  Read the argument with your task in mind.

The psychologist argues that psychiatrists should not resist prescribing antidepressant drugs to patients suffering from depression because most cases of depression are caused by a chemical imbalance and the drugs help eliminate such imbalances.

STEP 3:  Know what you’re looking for.

The only way to identify the one that is not a flaw is to review the answers one by one.

STEP 4:  Read every word of every answer choice.

Answer A is a flaw and cannot be your answer because the psychologist makes a categorical statement that doctors should stop refusing to prescribe the drugs altogether, which means that the psychologist makes no allowance for the fact that the drugs might be ineffective for some patients. Answer B is not a flaw because the psychologist does not say depression causes a chemical imbalance. The psychologist actually argues the opposite, so this must be your choice, but you should review the remaining options to be sure. Answer C can be considered a flaw for the same reason as answer A. The psychologist makes an unsupportable categorical statement that drugs should be prescribed for all. Answer D can be considered a flaw as well because the psychologist does assume that the chemical imbalances addressed by the drugs are a complete and known factor, and if this is not the case the psychologist has made an error in supporting the unrestrained prescription of such drugs. Finally, answer E can be considered a flaw. The psychologist assumes that drugs are the only way to treat the chemical imbalances, but if therapy can deal with them, then the psychologist’s argument that drugs are a complete solution falls apart. The correct choice is answer B.

5. Answer: D

STEP 1:  Read the question and identify your task.

This is a Paradox question. The question asks you to find among the answers the statement that resolves the discrepancy described in the argument.

STEP 2:  Read the argument with your task in mind.

The argument describes a situation in which an antidote to whooping cough has a good success rate but scientists warn that widespread use of the drug could be dangerous.

STEP 3:  Know what you’re looking for.

You expect that the correct answer will identify that widespread use versus limited use affects the safety of the drug in some way.

STEP 4:  Read every word of every answer choice.

Answer A cannot be the correct choice because the researchers warn about widespread use, not overuse. Nothing in the argument indicates a risk of incorrect dosages. Answer B assumes that the drug is supposed to be something more than an antidote and actually prevent the spread of the disease, which it is not. The discrepancy cannot be a result that is not expected of the drug in the first place. This cannot be your answer. Answer C may explain why the drug might be difficult to administer, but it does not indicate why it is dangerous even when effective as an antidote. Answer D offers a valid explanation for the discrepancy. Even if the drug is effective, if its use eventually results in the development of deadlier strains of the disease, then the drug could very well be too dangerous. This is most likely your answer, but you have one more option to consider. Answer E is an administrative concern but not one that would indicate why the drug is dangerous even if effective. The correct choice is answer D.

6. Answer: C

STEP 1:  Read the question and identify your task.

This is a Deduction question. The question asks that you identify the basis for the disagreement between Juan and Dorothy.

STEP 2:  Read the argument with your task in mind.

Juan argues that managers should adjust their managerial style to the employees, and Dorothy believes that employees should be flexible enough to adjust to the company’s needs. It could be said that the disagreement is over whether management should adjust to employees or employees should adjust to management.

STEP 3:  Know what you’re looking for.

You expect the correct answer to be similar to one of those two options.

STEP 4:  Read every word of every answer choice.

Answer A is problematic because it uses the word sometimes. Neither Juan nor Dorothy speaks in such half-terms. Answer B cannot be correct because their argument is not about training. Answer C says all employees should learn to adapt to managerial styles. This is similar to the answer you formulated and is most likely your choice. You should review the remaining options to be sure. Answers D and E cannot be correct because their argument is not over the nature of employees’ projects. Rather it is over the interaction between employees and managers. The correct choice is answer C.

7. Answer: C

STEP 1:  Read the question and identify your task.

This is a Conclusion question. The question asks you to choose the one statement that is supported by the political consultant’s argument.

STEP 2:  Read the argument with your task in mind.

The political consultant states that because politicians’ speeches are closely analyzed, politicians give consistent speeches that are not explicit on details.

STEP 3:  Know what you’re looking for.

You expect the correct answer to state something to the effect that people who are closely analyzed make consistent speeches with little detail.

STEP 4:  Read every word of every answer choice.

Answer A cannot be correct because the consultant does not say that politicians are the only people who are scrutinized in this way. Answer B cannot be correct because the consultant does not say politicians give the most consistent speeches. The consultant makes no comparison between politicians and other speech-givers. Answer C says that people give very consistent statements when closely analyzed. This fits your expected answer and is supported by the political consultant’s statements. This is most likely your answer, but you should review your remaining options. Answer D states the opposite of what the consultant is arguing by saying that people make consistent statements regardless of scrutiny. Answer E cannot be your answer mainly due to wording. It is uncertain whether “evaluated more strictly” means the same thing as “judged for consistency,” and “majority of things” is a somewhat vague term for comparison. The correct choice is answer C.

8. Answer: E

STEP 1:  Read the question and identify your task.

This is a Flaw question. The question asks you to identify the flaw in the media consultant’s argument.

STEP 2:  Read the argument with your task in mind.

The media consultant argues that electronic media is bringing an end to traditional news organizations. The consultant implies that the low cost and ease of publishing and distributing a newspaper no longer exist, and thus electronic media will make traditional news organizations obsolete.

STEP 3:  Know what you’re looking for.

You expect the correct answer to point out that just because the low cost and ease of publishing and distribution do not exist, that does not necessarily mean that traditional news organizations will cease to exist.

STEP 4:  Read every word of every answer choice.

Answer A cannot be correct because the consultant never makes use of an expert opinion in supporting the argument. Answer B cannot be correct because the consultant does not make an assessment regarding the value of a traditional news organization. At issue is whether another industry will put it out of business, not whether the value of that new industry will surpass the value of the traditional news organization. Answer C cannot be correct because the consultant is not presupposing that traditional news organizations will die out. The consultant is attempting to prove it by pointing out conditions that exist in the industry (more costly and difficult to publish, takeover by electronic media, etc.). Answer D is incorrect because the consultant is consistent in logical terms. The words “are bound to” are almost exactly the same as “inevitable,” and there is no inconsistency that is similar to saying that something will happen based on what might happen. Finally, answer E states that the consultant mistakes something that enables an institution to arise for something necessary for that institution’s survival. This is the flaw you expected. The consultant assumes that just because circumstances such as low cost and ease of publishing and distributing the newspaper no longer exist, traditional news organizations will die out. But it is possible that these organizations can find another way to survive, such as adopting the tools of electronic media, for example. The correct choice is answer E.

9. Answer: E

STEP 1:  Read the question and identify your task.

This is a Describe question. The question asks that you identify the statement that most accurately describes Lucie’s criticism of David’s statement.

STEP 2:  Read the argument with your task in mind.

David states that the effort to create new biofuels has caused an increase in demand for crops usually used for food. The resulting price increase for food is causing hunger and instability throughout the developing world, so he argues that advanced nations should scale back their efforts. Lucie criticizes his argument by citing factors other than biofuels that could be causing the increased prices that produced the problems.

STEP 3:  Know what you’re looking for.

The correct answer will describe Lucie’s logic as citing other factors that undermine David’s assumptions.

STEP 4:  Read every word of every answer choice.

Answer A cannot be correct because Lucie does not point out any contradictions in David’s statements. In fact, she does not exactly question his underlying facts, only the connection he draws between them. Answer B cannot be correct because Lucie brings up no such analogy. The crops in Russia and China and the meal bug infestation are not analogous to the demand for biofuels putting price pressure on food crops since the former are disasters and the latter is a market demand occurrence. Answer C cannot be your answer because Lucie’s argument is not supportable by any of David’s facts. Again, the difference between a market occurrence and a disaster prohibits this sort of logical flexibility. Answer D is incorrect because Lucie’s facts are not in conflict with David’s facts, so they can be easily reconciled since they are concerning different phenomena. Finally, answer E is your correct answer because Lucie does not question David’s facts. She only points out that his conclusion does not follow so easily from them. She thinks that his assumption that the development of biofuels is responsible for the increase in food prices is questionable. The correct choice is answer E.

10. Answer: C

STEP 1:  Read the question and identify your task.

This is a Weaken question. The question asks that you choose the statement that David might use to counter Lucie’s objection.

STEP 2:  Read the argument with your task in mind.

David states that the effort to create new biofuels has caused an increase in demand for crops usually used for food. The resulting price increase for food is causing hunger and instability throughout the developing world, so he argues that advanced nations should scale back their efforts. Lucie criticizes his argument by citing factors other than biofuels that could be causing the increased prices that resulted in the problems.

STEP 3:  Know what you’re looking for.

You expect that David would somehow prove that the disasters that affected the crops were inconsequential to the price increases that caused all the problems in the developing world.

STEP 4:  Read every word of every answer choice.

Answer A might be true, but the concern was the worldwide price increase, not just that affecting biofuels. This statement does not change the fact that the disasters in those countries could have affected the overall world crop supplies. Answer B has the same problem as answer A. The issue is food supplies, not just supplies for biofuels. Answer C is exactly what you are looking for. It says that the disasters are nothing new and could therefore be considered an inconsequential factor in the rise of world food prices. This is most likely your answer, but you must review your last two options. Answer D is an irrelevant statement since it is future-oriented and has nothing to do with the current increase in prices. Answer E is actually helpful to Lucie’s argument by giving another statement that undercuts David’s dependence on the development of biofuels as the reason for the price increases. You are looking for a statement to undercut Lucie, not help her, so the correct choice is answer C.

11. Answer: A

STEP 1:  Read the question and identify your task.

This is a Paradox question. The question asks that you identify the answer that does not help resolve a discrepancy between a report and some safety records. This means that four out of the five possible answers resolve the discrepancy.

STEP 2:  Read the argument with your task in mind.

The argument discusses a safety report indicating that a reduction in traffic signs leads to a decline in automobile accidents, but a certain city reduced its traffic signs by 50 percent and saw no decline in automobile accidents over a three-year period.

STEP 3:  Know what you’re looking for.

You must go answer by answer and test whether each one resolves the discrepancy.

STEP 4:  Read every word of every answer choice.

Answer A explains the accidents that occurred but does not explain why the decrease in signs did not have the desired effect of lowering the number of accidents. This is most likely your answer, but you should review the remaining options to be certain. Answer B indicates that bureaucratic errors resulted in the same number of traffic signs as before. Therefore, the reduction of 50 percent was misleading and you should not have expected the reduction in accidents to occur. This explains the discrepancy. Answer C indicates that the number of automobile accidents actually did decrease but due to the addition of a new category of accidents the total number of accidents is hiding a result that is actually consistent with the safety report. This explains the discrepancy. Answer D says there was an increase in the speed limit, which is another change in traffic conditions that could alter or change the results. You do not know, but it definitely makes the city’s results suspect and gives at least a partial explanation of the discrepancy. Answer E is another change in traffic conditions that could explain the increase in accidents. This also gives at least a partial explanation for the discrepancy. The correct choice is answer A.

12. Answer: A

STEP 1:  Read the question and identify your task.

This is a Flaw question. The question asks you to identify the error in reasoning in the argument.

STEP 2:  Read the argument with your task in mind.

The argument states that Jack said he was not going to include the marketing department in a meeting, but people left the meeting with a report that the marketing department had promised to produce and deliver at the meeting, so at least some members of the marketing department must have been at the meeting.

STEP 3:  Know what you’re looking for.

You might guess that the answer will identify the flaw as the assumption that the marketing department is the only department that could produce that particular sales report.

STEP 4:  Read every word of every answer choice.

Answer A is a close approximation of that exact flaw. It says that the argument fails to establish that something which is true for some people is true for only those people. Just because the marketing department promised to produce and deliver the report does not mean they are the only people who could produce and deliver the report. Someone else at the meeting could have fulfilled Jack’s need. This is most likely your answer, but you must review the remaining options. Answer B cannot be correct because the only such term used in the argument is the importance Jack put on the report, but this is not a flaw and is irrelevant to the logic of the argument. Answer C ignores the basis for the conclusion, that the report was delivered, and focuses on whether someone has a right to be at the meeting, which is an irrelevant factor, since you do not know whether or not the marketing department was at the meeting in the first place. Answer D is about Jack’s interest in the report, but whether or not he wanted the report is unimportant since the reasoning is based on the fact that the report was produced and delivered at the meeting, so this cannot be your answer. Finally, answer E says that the flaw is that Jack could have changed his mind based on changing circumstances, but you have no evidence of that change. The correct choice is answer A.

13. Answer: B

STEP 1:  Read the question and identify your task.

This is a Conclusion question. The question asks that you identify a statement that must be true based on the statements in the argument.

STEP 2:  Read the argument with your task in mind.

The argument is a series of “if . . . then” statements. Since the statements are in the negative, it might be helpful to create the contrapositive. The first statement: if you can repair the water heater, then you must be skilled with tools. The second statement: if you can perform the duties of a maintenance engineer, then you must be able to repair a water heater. Since both statements share a term (repair a water heater), you can also say that if you can perform the duties of a maintenance engineer, then you must be skilled with tools.

STEP 3:  Know what you’re looking for.

The correct answer will say that if you can perform the duties of a maintenance engineer, then you must be skilled with tools.

STEP 4:  Read every word of every answer choice.

Answer A cannot be your answer because you cannot reverse the second statement. “If A then B” does not mean “if B then A.” Answer B is exactly your contrapositive and must be your answer, but you should review the remaining options to be sure. Answer C has the same problem as answer A. You cannot reverse the contrapositive “if . . . then” statement and expect it to be true. Answer D adds the word probably to the statement, which changes the meaning of the statement and makes it unworkable with your logical statements. Concerning answer E, you must consider joining the statements to say that if you have no skill with tools at all, then you will not be able to perform the duties of a maintenance engineer. But once again, the reverse of this statement is not true, and this is why answer E cannot be correct. The correct choice is answer B.

14. Answer: D

STEP 1:  Read the question and identify your task.

This is an Assumption question. The question asks that you choose from among the answers the assumption the artist requires to make the argument.

STEP 2:  Read the argument with your task in mind.

The artist argues essentially that the meaning of a work of art is variable and that the intentions of the artist are irrelevant. Therefore, the interpretation reveals more about the critic than the artist.

STEP 3:  Know what you’re looking for.

You expect the correct answer will link the meaning of a work of art directly to the critic or interpreter.

STEP 4:  Read every word of every answer choice.

Answer A cannot be correct because it links understanding of the art to the artist’s history, which is in direct conflict with the statements saying that the artist’s intentions are irrelevant to an interpretation. Answer B cannot be correct because it states that the interpreter can never know the real intentions of the artist, but the art critic does not base the argument on not being able to know the artist’s intentions. Rather, the critic feels that those intentions are irrelevant to any interpretation. Answer C is similar to answer A in that it expresses the opposite of what the critic is arguing. Answer D makes a direct connection between the meaning of the work of art and the interpreter. This is the assumption as you formulated it and is likely the right answer, but you have one more option to consider. Answer E might be something the critic would say at the end of the argument, but nothing in the statements before you show that the critic is seeking to establish the validity of interpretations. Rather, the critic is making an observation about the nature of interpretations of works of art. This cannot be your answer. The correct choice is answer D.

15. Answer: D

STEP 1:  Read the question and identify your task.

This is a Parallel question. The question asks you to identify which of the answers follows the same pattern of reasoning as the argument.

STEP 2:  Read the argument with your task in mind.

The argument says that a study proves that replacing an older power plant with a newer one reduces the incidence of major illnesses but does not distinguish between conventional and nuclear power plants. Therefore, the argument claims, there is no increased health risk from living next to a nuclear power plant. The logic works off the fact that the study fails to distinguish between two types of subjects, and then makes a dubious conclusion based on that failure.

STEP 3:  Know what you’re looking for.

The correct answer will follow the same pattern.

STEP 4:  Read every word of every answer choice.

Answer A is straightforward concerning the two types of bags, but it does not base its conclusion on a failure to distinguish between two types of bags. It merely states its facts and derives a conclusion based on those facts. This does not follow the same pattern of reasoning and cannot be your answer. Answer B also features a study with no failure to distinguish between types. The study gives a choice between supplements and eating fruits and vegetables, and the statement says you can choose the latter over the former without health risk. This does not follow your pattern. Answer C makes a dubious claim based on questionable evidence but not on any failure to distinguish between two types of drivers. This cannot be your answer. Answer D features a study claiming that fruits and vegetables help reduce the incidence of cancer and says this benefit exists regardless of whether they are grown conventionally or organically. It then makes a dubious claim that there is no risk of increased cancer from eating fruits and vegetables containing pesticide residues. The claim is based on the fact that the study does not distinguish between the types of fruits and vegetables. This follows your pattern and is most likely your answer, but you have one more option to consider. Answer E does not base its claim on a failure to distinguish between genetic types. The dubious claim or leap in logic is based on a simple link between health and a genetic predisposition to eat an unhealthy diet. Therefore, this does not follow your pattern. The correct choice is answer D.

16. Answer: A

STEP 1:  Read the question and identify your task.

This is an Inference question. The question asks you to choose the statement that expresses a probable belief held by the league director based on his statements.

STEP 2:  Read the argument with your task in mind.

Mr. Anderson presents the situation, saying that his daughter’s entire soccer team was suspended because some members of the team were taunting members of the opposing team. He is angry because his daughter was not one of those doing the taunting, and everyone knows the culprits. The league director defends the decision to suspend the entire team by using an analogy to passengers forced to endure a delayed flight even though they had nothing to do with the reason the plane is delayed. It is obvious that the league director believes that Mr. Anderson’s daughter is innocent but should be punished along with the others anyway.

STEP 3:  Know what you’re looking for.

The correct answer will indicate the league director’s belief in the innocence of Mr. Anderson’s daughter.

STEP 4:  Read every word of every answer choice.

Answer A says the league director believes Mr. Anderson’s daughter might be innocent, and this is exactly what is implied by his statements since he compares her to an airline passenger who is delayed even though he or she had nothing to do with the delay. This is most likely your answer, but you must review the remaining options to be certain. Answer B focuses on an irrelevant aspect of the director’s argument, the causes of flight delays. While he might believe in this statement, there is nothing in the argument that leads us to infer that he understands why a flight is delayed. He only knows that the passengers are not responsible. Answer C cannot be correct because it is Mr. Anderson who says “everyone” knows who taunted them, which you can assume includes his daughter, but the league director does not suggest such knowledge, and the question asks what can be inferred only from the league director’s statements, not from Mr. Anderson’s. Answer D cannot be correct because the league director does not indicate what result he hopes to achieve from the punishment. He only attempts to justify the nature of the punishment. Finally, answer E cannot be correct because the league director’s statements suggest only that he believes that many team members did not taunt. His statements imply this by saying that some passengers on the plane were blameless for the delay. He gives no hint regarding his idea of the number of those who actually taunted the opposing team, or in accordance with the analogy, those people responsible for the delay. The correct choice is answer A.

17. Answer: E

STEP 1:  Read the question and identify your task.

This is a Weaken question. The question asks that you identify a vulnerability in the league director’s statements.

STEP 2:  Read the argument with your task in mind.

Mr. Anderson presents the situation, saying that his daughter’s entire soccer team was suspended because some members of the team taunted members of the opposing team. He is angry because his daughter was not one of those doing the taunting, and everyone knows the culprits. The league director defends the decision to suspend the entire team by using an analogy to passengers forced to endure a delayed flight even though they had nothing to do with the reason the plane is delayed.

STEP 3:  Know what you’re looking for.

The correct answer will be based on the flaw in comparing the punishment, which the league director may choose to impose or not, to the flight delay, which is unavoidable.

STEP 4:  Read every word of every answer choice.

Answer A cannot be your answer because the director does not discuss any facts about the incident. Answer B cannot be correct because he makes no such generalization regarding the entire team. He does not discuss the team at all. Answer C cannot be correct because the director compares Mr. Anderson’s daughter to airline customers who are blameless for the delay, so he must consider her blameless as well. Answer D cannot be your answer because it is not the taunting that is being compared to the inconvenience. It is the punishment that is being compared to the inconvenience. Finally, answer E says that the complaint does not acknowledge that the inconvenience of a flight delay is unavoidable, but the director could have chosen not to impose the punishment. This is a strong criticism of the director’s argument, so the correct choice is answer E.

18. Answer: A

STEP 1:  Read the question and identify your task.

This is a Weaken question. The question asks you to choose the statement that most weakens the editorialist’s argument.

STEP 2:  Read the argument with your task in mind.

The editorialist argues that the frontal lobe in teenagers is not well developed and adversely affects their decision making, which is why they make bad decisions while driving. Thus, additional restrictions should be put on teenage drivers.

STEP 3:  Know what you’re looking for.

You might expect the correct answer to either debunk the idea that teenagers have an underdeveloped frontal lobe or to argue that such a deficiency has no link to poor driving behavior.

STEP 4:  Read every word of every answer choice.

Answer A says that most automobile accidents can be attributed to the poor functioning of a part of the brain other than the frontal lobe. This would suggest that a poorly developed frontal lobe might have nothing to do with poor driving, and this could very well be your answer. You must review the remaining options before settling on this answer as your choice. Answer B says that other parts of the brain contribute to decision making in teenagers and adults, but this still does not negate the idea that the frontal lobe, when poorly developed, may result in bad decision making and poor driving behavior. This cannot be your answer. Answer C is incorrect because the argument uses the term “developed,” not “smaller.” To say that a teenager’s frontal lobe is not as well developed could mean something other than size, so saying that a teenager’s frontal lobe is similar in size to that of an adult does not necessarily mean that it is equally developed. Answer D cannot be your answer because it discusses a different lobe, and it is possible that some lobes develop more quickly than others. Finally, answer E cannot be correct because it is comparing adults to adults while the argument is comparing teenagers to adults. Logically, this cannot serve to weaken the argument. The correct choice is answer A.

19. Answer: B

STEP 1:  Read the question and identify your task.

This is a Parallel question. The question asks you to identify which statement most closely resembles the argument.

STEP 2:  Read the argument with your task in mind.

The argument states that the increased availability of electronic books has caused a decrease in sales of printed books. The logic is that an increased availability of a related product is causing the decrease in demand for another product.

STEP 3:  Know what you’re looking for.

The correct answer will follow the same pattern.

STEP 4:  Read every word of every answer choice.

Answer A does not conform to your model because it says the increase of one related product causes the increase of another product (increase-increase). Answer B says that the introduction of a new product has caused the decrease of an older related product. This does conform to your model since the increase of one causes the decrease of the other. This is likely your choice, but you must review the remaining options. Answer C does not conform because it says that two products are both insufficient to dominate the market. You have no idea whether the increase of one is causing the decrease or increase of the other. Answer D does not conform because it says that when a child cannot do one activity, the child does more of another activity. A decrease leads to an increase, which is not the same as your model argument. Finally, answer E does not conform because it says that a decrease in the use of union labor leads to an increase in the use of robots. A decrease leads to an increase, which is similar to answer D and cannot be your choice. The correct choice is answer B.

20. Answer: C

STEP 1:  Read the question and identify your task.

This is a Conclusion question. The question asks that you identify George’s main conclusion.

STEP 2:  Read the argument with your task in mind.

George argues that Kristen was wrong to tell their boss that they do not have the talent and resources to complete the project because they do have the talent and resources. George believes that Kristen told this to the boss because she did not want the project to succeed.

STEP 3:  Know what you’re looking for.

The correct answer will state something to the effect that Kristen should not have said what she did for the reasons she gave.

STEP 4:  Read every word of every answer choice.

Answer A cannot be George’s conclusion. George says that Kristen’s lie in this situation was wrong, but you cannot be sure he does not believe that lying in other cases is not warranted. Answer B focuses on the last sentence, expecting you to assume that the last sentence is the conclusion, but it is not. The last sentence is what George believes is the reason for Kristen’s lie and is meant to support his conclusion. George’s argument is intended to influence you regarding Kristen’s lie, not to inform you why she lied. Answer C states the conclusion George wants you to come to. He wants you to believe that it was wrong for Kristen to tell their boss that the project would fail due to a lack of talent and resources. This is likely your answer, but you should review the remaining options to be certain. Answer D cannot be correct because Kristen is not blaming the failure on the deficiencies. She is actually predicting the failure. Finally, answer E cannot be your choice because it leaves out the ethical judgment George is making. The correct choice is answer C.

21. Answer: B

STEP 1:  Read the question and identify your task.

This is a Flaw question. The question asks you to identify the statement that reveals a vulnerability in the justification George uses to make his judgment.

STEP 2:  Read the argument with your task in mind.

George argues that Kristen was wrong to tell their boss that they do not have the talent and resources to complete the project because they do have the talent and resources. George believes that Kristen told this to the boss because she did not want the project to succeed.

STEP 3:  Know what you’re looking for.

The correct answer will attack the general statement regarding morality that George uses to justify his argument.

STEP 4:  Read every word of every answer choice.

Answer A cannot be correct because George does not invoke pity upon Kristen. If anything, George is morally indignant toward Kristen. Answer B refers to the statement “saying such falsehoods can never be other than morally wrong . . . ,” which is indeed a general principle that needs further justification than the judgment regarding Kristen’s actions. This is most likely your answer, but you must review the remaining options. Answer C cannot be correct because surely what Kristen says concerning the project is within her control and can be judged by such a moral standard. Answer D cannot be correct because you do not know (and George does not say) what Kristen knows or does not know with regard to the talent and resources of the company, so George’s justification cannot be judged vulnerable on this basis. Answer E cannot be correct because no cause and effect is being addressed in the argument. The entire argument concerns a prediction of what might happen, not what did happen. The correct choice is answer B.

22. Answer: D

STEP 1:  Read the question and identify your task.

This is a Flaw question. The question asks you to identify the flaw in the argument.

STEP 2:  Read the argument with your task in mind.

The argument says that members of the police force were raised in economically disadvantaged households and bases this conclusion on the fact that the communities in which they grew up had household incomes lower than the average household income for the nation as a whole.

STEP 3:  Know what you’re looking for.

You would expect the correct answer to note that just because the communities in which the police officers grew up had a lower average household income than the nation as a whole, this does not necessarily mean that the individual families of the police officers had a lower household income than the nation as a whole.

STEP 4:  Read every word of every answer choice.

Answer A cannot be correct because the argument concerns the incomes of the households in which the officers grew up, not their current household income. Answer B cannot be correct because it simply clarifies the basis for the argument by describing the type of neighborhood without pointing out a flaw in that description. Answer C cannot be your answer because the argument concerns the police officers and where they grew up, but it is not concerned with how many communities facilitated the rearing of future police officers. Answer D points out that the argument assumes that within their communities the officers lived in households with average or below-average income. This is exactly the problem with the argument. If the police officers grew up in households with above-average (or even the highest) incomes for their communities, then the fact that the community has below-average household income compared to the country is no longer relevant and it is possible they grew up in economically advantaged households. This is most likely your answer, but you have one more option to consider. Answer E cannot be correct because it raises a broader issue than the argument at hand. It is true that other factors like debt and assets might affect a household’s economic situation, but household income is the logical basis the argument uses for economic advantage or disadvantage. Either way, answer E is not as strong as answer D. The correct choice is answer D.

23. Answer: B

STEP 1:  Read the question and identify your task.

This is a Conclusion question. The question asks you to identify the statement that must be true given the statements in the argument.

STEP 2:  Read the argument with your task in mind.

The argument makes several assertions regarding astronauts, age, and purchasing alcohol, and you must determine whether they support one of the possible answers. The statements can be simplified as follows: astronauts > 21; purchase alcohol > 21; some scientists are astronauts; some scientists purchase alcohol; and some scientists < 21.

STEP 3:  Know what you’re looking for.

The correct answer will be consistent with the logical statements stated in step 2.

STEP 4:  Read every word of every answer choice.

Answer A cannot be supported by the statements because you know only that some scientists are astronauts. None of the statements says how many astronauts are scientists. Answer B is supported by the statements. If only some of the scientists are astronauts and only some of the scientists purchase alcohol, then it logically follows that some scientists are not astronauts and some astronauts do not purchase alcohol. This is most likely your answer, but you must review the remaining options. Answer C cannot be correct because none of the statements reveals a relationship between those who purchase alcohol and astronauts. Without such a statement, you cannot come to this conclusion. Answer D cannot be true because the statements tell you that some scientists are not astronauts, some do not purchase alcohol, and some are over the age of twenty-one. Therefore, it is possible that there is at least one scientist who is not an astronaut, does not purchase alcohol, and is over twenty-one. Finally, answer E cannot be correct because the first statement says that there are no astronauts under the age of twenty-one, but this does not negate the possibility that there is an astronaut who is exactly twenty-one years old. The statement would have to say that there are no astronauts aged twenty-one or younger, but this is not the case. The correct choice is answer B.

24. Answer: C

STEP 1:  Read the question and identify your task.

This is a Paradox question. The question asks you to identify the answer that resolves a discrepancy in the argument.

STEP 2:  Read the argument with your task in mind.

The argument discusses how the spread of kudzu and other invasive plant species has posed a threat to biodiversity, but then the argument says that scientific studies show that invasive plant species are rarely the cause of native species’ extinctions.

STEP 3:  Know what you’re looking for.

You expect the correct answer to point out that while kudzu and other such invasive species threaten the biodiversity in local areas, they do not threaten the overall extinction of species.

STEP 4:  Read every word of every answer choice.

Answer A may affect how you value the importance of the discussion by devaluing the threat to biodiversity, but it does nothing to resolve the discrepancy. Answer B might be seen as a benefit of kudzu when compared to other invasive plants, but this statement does not resolve the question of why kudzu affects local biodiversity but does not cause species’ extinctions. Answer C resolves the discrepancy by saying that kudzu can affect local biodiversity while not threatening the existence of species on a wider scale. This is most likely your answer, but you must review the remaining options to be sure there is not a better one. Answer D may be true, but it does not negate the existing damage caused by invasive species, nor does it resolve the discrepancy in the facts at hand now. Answer E explains the extinctions, but it does not address why kudzu is not included in their list of causes even though it does so much damage to local biodiversity. The correct choice is answer C.

25. Answer: C

STEP 1:  Read the question and identify your task.

This is a Flaw question. The question asks you to identify a logical error in the scholarly review.

STEP 2:  Read the argument with your task in mind.

The author takes issue with M’s paper and its claims based on M’s tendency to ascribe bad faith to scholars who disagree with her and the distasteful nature of M’s personality. Based on those observations, the author recommends that M’s paper should be ignored by scholars.

STEP 3:  Know what you’re looking for.

You expect that the correct answer will point out the problem with basing criticism of the paper on M’s past behavior and M’s personality.

STEP 4:  Read every word of every answer choice.

Answer A cannot be correct because there is no indication that the author’s account of M’s criticism is biased in any way. It is very possible that the author’s recounting of M’s criticism is accurate, so this cannot be the error. Answer B cannot be the error because the author does not question the veracity of M’s criticism. The author merely comments on the significance of M’s criticism given the nature of the source. Regardless of whether the criticism is true or not, it seems that the author would still argue that the paper does not merit attention. Answer C is exactly what you expected to be the correct answer. The author uses an attack on M’s character as evidence that M is not competent as a scholar and that M’s criticism does not merit attention from “serious scholars.” This is likely your answer, but you must review the remaining options to be certain of your choice. Answer D cannot be the error because the author makes no assertions concerning the paper. The author merely mentions the substance of M’s criticism. Answer E cannot be your answer because it is more likely an error that M would commit if M argued a point based on the paper. The correct choice is answer C.

26. Answer: E

STEP 1:  Read the question and identify your task.

This is a Describe question. It asks you to choose the statement that describes the role of the first sentence in the argument.

STEP 2:  Read the argument with your task in mind.

The argument states that regulatory regimens are created to make sure government services are delivered fairly. Thus, despite qualms with the system, it is unlikely the government will simplify regulations. The first sentence seems to be a supporting fact that justifies the conclusion.

STEP 3:  Know what you’re looking for.

The correct answer will offer a supporting fact or premise that justifies the conclusion that regulations are unlikely to be simplified.

STEP 4:  Read every word of every answer choice.

Answer A cannot be correct because the conclusion that regulations will not be simplified is based on the fact that they are created as they are in order to ensure fairness. The statement does not weaken the conclusion. Answer B cannot be true because the word thus tells you that it is the first sentence that leads the author to conclude the second sentence, not the other way around. Answer C cannot be correct because instituting fairness does not justify the complexity of the regulations. It justifies why they will remain complex and not be simplified. Answer D cannot be correct because the phrase “growing dissatisfaction with complex regulatory systems” does not support the fairness in regulatory systems. If anything, it is the one statement that controverts that effort. Finally, answer E is the one premise that supports the claim that it is unlikely things will change. The effort for fairness supports the conclusion that regulations will not be simplified. The correct choice is answer E.

27. Answer: A

STEP 1:  Read the question and identify your task.

This is an Assumption question. The question asks you to choose the answer that gives an assumption upon which the argument depends.

STEP 2:  Read the argument with your task in mind.

The argument claims that presidents cannot achieve greatness by remaining in the capital city and that they must go out and immerse themselves in communities around the country in order to gain an understanding of the citizens’ everyday experiences.

STEP 3:  Know what you’re looking for.

You can guess that the correct answer will link a president’s greatness to gaining an understanding of a citizen’s everyday experiences.

STEP 4:  Read every word of every answer choice.

Luckily, answer A is right on the mark. It says that presidents cannot achieve greatness without an intuitive grasp of a citizen’s everyday experiences and frustrations. This is exactly the assumption that supports the argument and it is likely your answer, but you must review the remaining options to be certain. Answer B adds details to the interaction a president may participate in while immersed in outside communities, but it is not the participation that will make the president great but the understanding the president obtains through that participation, so this cannot be your answer. Answer C says the skills the president hones while in the capital city are necessary for greatness, but the argument says these are useful but not enough for greatness, so this cannot be your answer. Answer D cannot be correct because the argument says nothing about life experience. Its statements revolve around activities while in office. Finally, answer E cannot be the correct choice because the argument does not say what approach or methodology the president must take when immersed in communities, so the president could remain impartial or get actively involved in issues. The correct choice is answer A.